Главная Юзердоски Каталог Трекер NSFW Настройки

Математика

Ответить в тред Ответить в тред
Check this out!
<<
Назад | Вниз | Каталог | Обновить | Автообновление | 501 85 250
Общий тред Аноним 06/02/17 Пнд 19:56:45 10487 1
logo32.png 47Кб, 354x251
354x251
Это - тред общематематических разговоров. Он призван выполнять те функции, которые стихийно выполнял тред для начинающих, и, ранее, общий тред в /sci. Если хочется просто о чем-нибудь поговорить - пишите сюда. Если ваш вопрос достаточно конкретен - лучше воспользуйтесь одним из тематических тредов.
Аноним 10/02/22 Чтв 15:39:05 93720 2
>>93715
я не стану в этом участвовать
Аноним 10/02/22 Чтв 19:16:30 93735 3
ice.png 2008Кб, 1728x2148
1728x2148
Аноним 10/02/22 Чтв 20:40:09 93737 4
>>93735
Наверху Арифметика Серра
Аноним 10/02/22 Чтв 21:17:31 93741 5
>>93715
Вербицкий в своей программе по сути и написал в 10 году, у анона выйдет хуйня с "крутыми словечками из мемов" в нижнем уровне, типа ТНН или перфектоидов.
Аноним 11/02/22 Птн 08:36:18 93763 6
>>93735
Так, ну, куда-нибудь сверху надо влепить учебники Демидовича и Винберга.
Аноним 11/02/22 Птн 09:50:36 93764 7
image.png 946Кб, 384x1823
384x1823
image.png 1107Кб, 1200x3600
1200x3600
>>93705
>ОН БЫЛ БЛЯТЬ, его форсили с еще в наукаче в 2015-2016 годах.
Да. Был. Первый пик она.
Аноним 11/02/22 Птн 12:06:50 93772 8
>>93764
А логика типа не математика, почему ее разделы не отображены?
Аноним 11/02/22 Птн 12:16:47 93773 9
>>93772
>А логика типа не математика
Ну вообще, да. Я бы её отдельно от математики в Computer Science закинул.
Аноним 11/02/22 Птн 13:50:31 93775 10
>>93772
Да, логика - не математика. Это независимая научная дисциплина с другой областью исследований.
Аноним 11/02/22 Птн 14:45:11 93780 11
>>93773
>>93775
Математическая логика это математика, не несите хуйни пожалуйста.
11/02/22 Птн 15:48:10 93782 12
>>93780
Чел, даже математический анализ нематематика, а уж логика.
Аноним 11/02/22 Птн 16:17:26 93783 13
>>93782
Чем аргументируешь, тифареторебёнок?
Аноним 11/02/22 Птн 20:15:14 93796 14
>>93783
Математика несводима к логике. Это раньше люди так думали, что сводима, а она оказалась несводима.
Аноним 11/02/22 Птн 20:44:57 93799 15
>>93796
Я не утверждал что сводима, хоть я и частично не согласен с тем что не сводима, я утверждал что математическая логика это часть математики.
Аноним 11/02/22 Птн 20:54:58 93801 16
>>93799
Ну это ты сам придумал, что часть математики. Это крайне маргинальная точка зрения, даже более маргинальная, чем то что математика вообще сама по себе, а никакая мат логика там не нужна. Я считаю, что твоё утверждение ни истинное, ни ложное, а бессмысленное, поэтому и обсуждать тут нечего.
Аноним 11/02/22 Птн 21:05:25 93802 17
>>93801
Медаль Филдса, самую престижную математическую награду, дали Коэну за результат в математической логике. В самом престижном математическом журнале Annals of mathematics, в 2021 году была опубликована статья по математической логике (Martin’s Maximum++ implies Woodin’s axiom (∗)), мат.логиков общепринято называют математиками, скажем вот статья про Шелаха на mathshistory https://mathshistory.st-andrews.ac.uk/Biographies/Shelah/ первое предложение "Saharon Shelah is an Israeli mathematician who worked in America on mathematical logic", мат.логики сами себя называют математиками, скажем вот личная страничка Оливии Карамелло https://www.oliviacaramello.com/ первое предложение "I am a mathematician working as Associate Professor at the University of Insubria in Como.". Получается что точка зрения не маргинальная, а оччень даже общепринятая, а то что она маргинальная придумал сам себе ты, так как аргументов у тебя нету.
"
Аноним 11/02/22 Птн 21:21:38 93803 18
>>93802
Ну если слово употребляют именно с таким значением, то так оно и есть, потому что значение слова - это его употребление. А вообще существует предикатная логика, на её языке формулируется прикладная (для предикатной логики) теория - теория множеств. Затем на языке теории множеств формулируются содержательные математические понятия вроде функции. То есть уже прикладная теория по отношению к теории множеств. На языке математики в свою очередь тоже можно сформулировать прикладную теорию, например, для таких дисциплин как физика или экономика. То есть такая иерархия языков получается. Логику тоже можно попытаться содержательно описать в терминах семиотики, Фреге примерно что-то такое и сделал, когда провёл различие между интенсионалом и экстенсионалом.
Аноним 12/02/22 Суб 03:48:20 93816 19
0a461e0bd105a73[...].png 135Кб, 1140x604
1140x604
58d9fd7f106bb0a[...].png 98Кб, 1140x455
1140x455
a0c14f35a444ea6[...].png 113Кб, 1140x744
1140x744
>>93780
У логики и у математики различаются области исследования. Различаются инструменты исследования. Различаются результаты. Различается даже язык. Математик в здравом уме ничего подобного пикрелейтед не напишет, а для логиков такие тексты - обычное дело.
Кановей. Определимость с помощью степеней конструктивности. АН СССР, исследования по теории множеств и неклассическим логикам, Москва, Наука, 1976 г. Редакторы Бочвар, Гришин
Аноним 12/02/22 Суб 03:53:11 93817 20
>>93802
А есть ли примеры обратного - что какую-нибудь известную награду в области логики давали математику?

Самая известная награда достаётся неизменно логикам, а не математикам.
https://en.wikipedia.org/wiki/G%C3%B6del_Lecture
Аноним 12/02/22 Суб 04:50:09 93818 21
>>93816
Я за мат.логику говорю, не за логику вообще, что более широкое понятие. Область исследования мат.логики: это формальные языки, нормальные комбинаторные структуры, не хуже теории графов. Инструменты исследования те же самые: доказательство теорем рассуждениями. Результаты те же самые: теоремы об исследуемом объекте. Не обычное дело, открой arxiv Math.LO и посмотри статьи которые написали за последнюю неделю или месяц https://arxiv.org/list/math.LO/recent нормальный там язык.

>>93817
Я не согласен с тем что мат.логики не являются математиками, поэтому ответ: тривиально да.
Аноним 12/02/22 Суб 05:28:13 93819 22
2112.14606.pdf.png 134Кб, 729x636
729x636
formula2.PNG 111Кб, 612x713
612x713
formula3.PNG 106Кб, 599x599
599x599
formula4.PNG 127Кб, 598x524
598x524
>>93818
>нормальный там язык.
Статьи можно листать дальше абстракта.

>поэтому ответ
Ну, мне просто кажется сомнительным аргумент, что все, кто получал медаль Филдса, автоматически являются математиками. Эту медаль, например, Виттен получал. Мне кажется, иногда её дают не-математикам, если достижения кажутся интересными.
Аноним 12/02/22 Суб 05:36:53 93820 23
screen.png 329Кб, 1986x1282
1986x1282
>>93819
Статьи как статьи, ну нагруженные формулами куски, бывает, вот первая статья по general topology, не вижу чем это лучше или хуже.
>Ну, мне просто кажется сомнительным аргумент, что все, кто получал медаль Филдса, автоматически являются математиками. Эту медаль, например, Виттен получал. Мне кажется, иногда её дают не-математикам, если достижения кажутся интересными.
Мне не кажется, Виттен получил потому что Виттен в том числе и математик, и гениальный вообще говоря, даже по меркам филдсовского лауреата.
Аноним 12/02/22 Суб 05:49:02 93821 24
>>93820
Типографскую сложность сравни. Количество разных индексов, над- и подстрочных. Диакретику. Не-латинские символы.
Формулы логики отличаются и от формул математики, и от формул информатики, и от формул физики. С первого взгляда видно, где логик развивает исчисление, а где физик шатает лагранжианы.
Аноним 12/02/22 Суб 06:04:14 93822 25
>>93821
Это специфика области просто, у вероятности например тоже довольно специфический стилёк с их постоянными \mathbf E[XY] для матожидания. Короче я вообще не знаю нахуя я спорю, я не то чтобы не согласен что логика немного более выделена, просто говорить что это не математика это слишком резко и обидно за логиков стало. Причина разногласий просто в том что слово "математика" чувствуют все по разному, а определения по ГОСТу, слава богу, нет, поэтому каждый отстаивает легитимность своего ощущения, что бессмысленно почти всегда.
Аноним 12/02/22 Суб 06:37:56 93823 26
>>93822
Как будто если не математика, то обязательно ерунда. Ничего подобного же. Просто отдельная наука, смежная. Иногда занимается с математикой перекрестным опылением.
Аноним 12/02/22 Суб 15:30:04 93826 27
https://youtu.be/MhRE8VkT46w
Как математики к философии относятся и к философо-математическим вопросам? Должен ли философ знать математику, а математик философию? Вот на защите докторской по философии Родина сидит Вавилов и хвалит, что тот сечет в современной математике, не являясь при этом математиком. А я думал, математики философов просто балаболами считают.
Аноним 12/02/22 Суб 18:37:27 93828 28
>>93826
>Как математики к философии относятся и к философо-математическим вопросам?
По-разному. Есть которые интересуются, уважают и сами что-то пишут (Барри Мазур и Ален Конн, например), есть которые считают чистым балабольством.
>Должен ли философ знать математику
Если он философ математики или использует математику в своей работе, то да.
>а математик философию
Если он пытается на философские темы рассуждать, то да.
Аноним 12/02/22 Суб 21:25:00 93831 29
image.png 441Кб, 996x716
996x716
image.png 633Кб, 973x692
973x692
image.png 619Кб, 983x697
983x697
Вот оглавление примерно такого усреднённого учебника по логике. Насколько всё это имеет отношение к математике? Какое-то пересечение, безусловно, можно найти, но по большей части это всё таки не математика.
Аноним 12/02/22 Суб 21:57:57 93833 30
>>93764
большое тебе спасибо, благодаря тебе я вспомнил мои подростковые годы, когда я поливал деревья и листал еще первые математико треды в наукаче
Аноним 12/02/22 Суб 21:59:46 93834 31
>>93764
хах через два года буду на третьем уровне дай Бог
Аноним 25/02/22 Птн 22:28:19 94327 32
И как теперь математикам по международным конференциям ездить?
Аноним 26/02/22 Суб 02:26:20 94332 33
>>94327
уже два года не ездят
все конференции проходят онлайн
Аноним 26/02/22 Суб 04:47:42 94333 34
>>94332
Ничего, зато ICM22 не онлайн, хоть туда съездят.
Аноним 26/02/22 Суб 09:02:48 94334 35
>>94333
кстати, может и накрыться теперь
трудно представить, как оно пройдёт
наверно, будет, но со скандалами
Аноним 26/02/22 Суб 10:53:19 94335 36
image.png 834Кб, 710x1280
710x1280
Аноним 26/02/22 Суб 13:25:41 94336 37
>>94332
Так теперь на онлайн конференцию просто не пустят, любые контакты с учеными РФ будут запрещены.
Аноним 26/02/22 Суб 14:25:22 94337 38
>>94336
Нахуя панику разводить? Я видел мнения многих математиков в твиттере, общий посыл примерно такой - "мы не хотим поддерживать мероприятия, связанные с правительством РФ, не хотим, чтобы п-н вручал медали Филдса нам, но общаться с коллегами по зуму не против, это все не их вина"
Аноним 26/02/22 Суб 20:37:35 94341 39
image.png 142Кб, 632x549
632x549
>>94335
Ну, собственно, ожидаемо. Спасибо, съездил волонтером.
Аноним 26/02/22 Суб 23:12:37 94343 40
Представители Сколковского института науки и технологий (Сколтех) сообщили, что его сотрудничество с Массачусетским технологическим институтом (MIT) прекращается. 25 февраля MIT направил Сколтеху соответствующее уведомление, назвав причиной разрыв сотрудничества проведение Россией специальной операции на Украине.

https://www.mk.ru/politics/2022/02/26/massachusetskiy-institut-otkazalsya-rabotat-so-skolkovo.html
Аноним 26/02/22 Суб 23:52:47 94346 41
The Post Millennial
@TPostMillennial

Rep. Eric Swalwell suggested that "kicking every Russian student out of the United States" should be "on the table."
Аноним 28/02/22 Пнд 17:42:03 94361 42
>>94336
Сомневаюсь, что прекратят контакты по своей инициативе. Даже во времена худших витков холодной войны контактировали, правда тогда и средств не было соответствующих, и само советское правительство воспрещало. Здесь, думаю, российским математикам достаточно будет зачитать покаянно-унизительную речь про нетвойне и спокойно сидеть в зуме.
Аноним 28/02/22 Пнд 18:35:36 94362 43
ФРГ свернула сотрудничество с Россией в сфере образования

Германия отказывается от сотрудничества с Российской Федерацией в образовательной сфере. Об этом сообщили в министерстве образования ФРГ.

Берлин достаточно плотно сотрудничал с Москвой в сфере образования. В Германии учится большое количество российских студентов. Теперь у них могут возникнуть проблемы, вплоть до отчисления из университетов.

Ранее уполномоченный по правам человека при президенте РФ Татьяна Москалькова подняла вопрос о незаконных отчислениях российских студентов, обучающихся в вузах Франции, Бельгии, Чехии и ряда других европейских государств. После начала военной операции по демилитаризации и денацификации Украины молодых россиян в массовом порядке стали исключать из списков студентов европейских вузов.

https://www.mk.ru/politics/2022/02/28/frg-svernula-sotrudnichestvo-s-rossiey-v-sfere-obrazovaniya.html
04/03/22 Птн 10:53:51 94398 44
Так двач все же закроют?

Куда вы уйдете после этого? В мазстакэксенджс? 4chan - /sci? Неужели вы не хотите все куда нибудь перебраться?
Аноним 04/03/22 Птн 10:54:19 94399 45
за сажу простите
Аноним 04/03/22 Птн 13:42:19 94400 46
>>94398
>Куда вы уйдете после этого?
На dxdy, конечно же.
Аноним 04/03/22 Птн 14:03:54 94401 47
>>94398
Все уже ушли давно. На доске постят раз в два дня.
Аноним 06/03/22 Вск 23:23:42 94411 48
Исполком Международного математического союза на внеочередном заседании принял решение об исключении Союза математиков России из организации и приостановил членство в ней всех ученых, имеющих российское гражданство. Все теоремы, доказанные российскими и советскими математиками, и носящие их имена, подлежат переименованию.

Исполком представит свои предложения по тому, чьи имена следует присвоить российским теоремам, Международному математическому конгрессу, назначенному на июль 2022 года для утверждения.

Так, например, теорема Остроградского будет теперь именоваться теоремой Гаусса, уравнение Менделеева станет уравнением Клапейрона, неравенство Буняковского станет неравенством Шварца (или Коши), а условия д'Аламбера-Эйлера станут условиями Коши-Римана (Эйлер вторую часть своей жизни прожил в России, имел российское подданство и похоронен в Александро-Невской лавре в Санкт-Петербурге). Разногласия возникли по поводу знаменитой формулы Эйлера - пока непонятно, кому её приписать так, чтобы не возникло непонимания. Отмечается, что аналогичная сложность в 2010 году возникла с формулой Муавра (Каддафи), но тогда проблему удалось замять.

Российский математический союз уже выразил протест по поводу принятого решения.

"Как можно переименовывать теоремы - ведь авторство результата от этого не изменится! Если теорему доказал я, то уж не вы, и как это можно поменять?" заявил известный российский математик Алексей Савватеев.

Однако в Исполкоме указали на теорему Арнольда, гласящую, что теорема никогда не называется в честь открывшего её математика, так что проблемы нет. В исполкоме добавили, что теорему Арнольда также ожидает переименование.
Аноним 07/03/22 Пнд 22:49:35 94417 49
Аноним 11/03/22 Птн 19:08:33 94438 50
На сайте мцнмо пропала часть ссылок на книжки. Копирасты таки дотянулись?
Аноним 11/03/22 Птн 21:25:05 94442 51
>>94438
возможно, авторы
хуй знает
впереди тьма
Аноним 14/03/22 Пнд 12:24:23 94485 52
ssf24d20597a33a[...].jpg 33Кб, 600x337
600x337
Аноним 14/03/22 Пнд 14:02:26 94486 53
Аноним 18/04/22 Пнд 13:14:50 95293 54
По каким учебникам лучше вкатываться?

Школьную программу уже забыл.
Аноним 18/04/22 Пнд 17:35:49 95294 55
Аноним 18/04/22 Пнд 17:53:24 95295 56
Подскажите опытные математики, как вы прорабатываете новые для себя предметы/разделы/темы.
Допустим, решили изучить алгебраическую геометрию. Открыли на ютубе лекции Вавилова, посмотрели. Порешал выборочно задачи из задачника.
Всё на этом? Как понять, поняли ли тему? Нужна ли работа с учебником, когда уже появились видеолекции? И как комбинировать видеолекции и учебники, они ведь не слишком то стыкуются?
Аноним 18/04/22 Пнд 18:15:05 95296 57
VT.png 92Кб, 608x834
608x834
Купил книгу pdf. Если я её выложу на либген или еще куда, чем мне это грозит? Прост в интернете есть только 2002г версия. Какие меры предосторожности следует предпринять, чтобы меня не набутылили?
Аноним 18/04/22 Пнд 18:15:30 95297 58
VT.png 92Кб, 608x834
608x834
Купил книгу pdf. Если я её выложу на либген или еще куда, чем мне это грозит? Прост в интернете есть только 2002г версия. Какие меры предосторожности следует предпринять, чтобы меня не набутылили?
Аноним 18/04/22 Пнд 18:17:57 95298 59
я извеняюсь просто интернет мобильный в лесу плохо ловит поэтому так вышло...
Аноним 18/04/22 Пнд 22:16:34 95307 60
Gavr-3.jpg 125Кб, 448x700
448x700
>>95295
>Как понять, поняли ли тему
Построение некоторой онтологии предмета + список основанных на ней контрольных вопросов.
https://onlinelibrary.wiley.com/doi/abs/10.1207/s15516709cog0204_3
Вообще, про "понимание" очень много чего написано. Понимание тесно связано со знанием, которое не менее связано со структурированием информации, и при этом существуют десятки подходов как к осмыслению понятий структуры и информации, так и методам приложения одного к другому. Можно, например, трактовать понимание как построение личной эффективной СУБД. Или ограничить его умением отвечать на корректно сформулированные вопросы по теме. Или способностью сдать экзамен. Или - или, универсальных критериев нет.

>видеолекции
Ценность видеолекций заключается в наличии ряда облегчающих понимание паралингвистических элементов: занятных графиков, забавных картинок, значков и закорючек, многозначительных хмыканий, поясняющих жестов и глубокомысленных помаваний руками. Также важны управляющие процессом научения мета-элементы: диалог с аудиторией, риторические вопросы, комментарии к происходящему, повторы для закрепления, анекдоты и отступления для психологической разрядки и т. д.

Ну то есть видеолекции расширяют канал входа информации и давят на некоторые ускоряющие обучение кнопки в мозге. Но они никогда полностью не заменят учебник, т. к. гораздо менее связны, менее детальны и хуже структурированы. Учебники более полезны для изучения материала, а лекции - для его объяснения.
Аноним 19/04/22 Втр 01:20:44 95309 61
>>95297
ниче не грозит, всем пофигу
Аноним 19/04/22 Втр 08:59:54 95313 62
Почему в России все студенты так сильно боятся задавать вопросы?

>учишься в школе
>соседка по парте вечно просит тебя за неё спросить / задать вопрос (на матеше)
>приходишь в универ на мехмат
>сосед по парте вечно просит тебя за него задать вопрос на паре
>"почему?"
>"ой, ну, я просто стесняюсь)"

>никто задачу не сделал
>все при этом думают, что все остальные задачу сделали, поэтому не спрашивают
>препод: "вопрос по ДЗ нет, нуок, двигаемся даже"

Меня это так сильно бесит. У меня нет каких-то сильных психических загонов по поводу того, что я кажусь "тупым", задавая вопросы. Но почему я единственный в группе-то должен казаться дураком, когда это не так?

Аноним 19/04/22 Втр 09:22:45 95314 63
>>95313
На самом деле те, кто задают вопросы, никогда дураками не кажутся, даже если это действительно примитивные волосы (глупые - не то слово), даже если ответы на эти вопросы они долго не понимают и переспрашивают. Наоборот, те, кто молчат, отсиживаются на задних партах, никак себя не проявляя, вызывают неприятные подозрения и нежелание с ними работать

мимо препод
Аноним 19/04/22 Втр 12:56:57 95321 64
>>95313
откуда-то из Арнольда про Фейнмана, говорящего об образовании в Бразилии:
По словам Фейнмана, студенты эти ничего не понимают, но никогда не задают вопросов, делая вид, что понимают всё. А если кто-нибудь начинает задавать вопросы, то курс его быстро ставит на место, как зря отнимающего время у диктующего лекцию преподавателя и у записывающих её студентов. В результате никто не может ничего из выученного применить ни в одном примере. Экзамены же (догматические, вроде наших: сформулируйте определение, сформулируйте теорему) благополучно сдаются. Студенты приходят в состояние «самораспространяющейся псевдообразованности» и могут в дальнейшем подобным же образом учить следующие поколения. Но вся эта деятельность полностью бессмысленна и фактически наши выпуски специалистов в значительной мере являются обманом, липой и приписками: эти так называемые специалисты не в состоянии решить простейших задач, не владеют элементами своего ремесла.
Аноним 20/04/22 Срд 00:20:31 95334 65
Почему при выводе корней для квадратного уравнения, когда мы извлекаем в одном месте корень, мы не пишем делать на 2 модуля А, а просто опускаем модуль. Что за хуйня?
Аноним 26/04/22 Втр 13:01:31 95419 66
>>95334
Мне потребовался двухнедельный нофап, чтобы понять твой вопрос.
>а просто опускаем модуль
Он опускается за кулисами. В формуле не просто sqrt(D), а +-sqrt(D).
Аноним 27/04/22 Срд 19:59:48 95435 67
>>95321
Так ведь это же эксперимент с китайской комнатой, только уже не мысленный! Вот она, философия.
Аноним 28/04/22 Чтв 00:17:38 95442 68
Как у ультрафинитистов, отрицателей бесконечности и проч. объясняется теорема Пифагора для прямоугольного треугольника с катетами, равными единице? В том смысле, как они вообще объясняют смысл / существование / нахождение длины гипотенузы (корень из 2)?
Аноним 28/04/22 Чтв 12:12:23 95445 69
>>95442
Наверно у них пиксельные треугольники
Аноним 28/04/22 Чтв 13:34:04 95449 70
>>95442
В теореме Пифагора речь идёт о квадрате длины, что гораздо более мощно, а не о длине.
> Quadrance and spread are quadratic quantities, while distance and angle are almost, but not quite, linear ones. The quadratic view is more general and powerful. At some level, this is known by many mathematicians. When this insight is put firmly into practice, as it is here, a new foundation for mathematics and mathematics education arises which simplifies Euclidean and non-Euclidean geometries, changes our understanding of algebraic geometry, and often simplifies difficult practical problems

И на базе этого можно построить рациональную тригонометрию без придуманных иррациональных чисел.

> New laws now replace the Cosine law, the Sine law, and the dozens of other trigonometric formulas that often cause students difficulty. The most important new laws are the Triple quad formula, the Spread law, the Cross law and the Triple spread
formula. Pythagoras’ theorem, restated in terms of quadrances, also plays a key role. The derivation of these rules from first principles is straightforward, involving some moderate skill with basic algebra. The usual trigonometric functions, such as cos θ and
sin θ, play no role at all.

> Rational trigonometry deals with many practical problems in an easier and more elegant fashion than classical trigonometry, and often ends up with answers that are demonstrably more accurate. In fact rational trigonometry is so elementary that almost all calculations may be done by hand. Tables or calculators are not necessary, although the latter certainly speed up computations. It is a shame that this theory was not discovered earlier, since accurate tables were for many centuries not widely available.

http://www.ms.lt/derlius/WildbergerDivineProportions.pdf
Аноним 02/05/22 Пнд 18:21:29 95590 71
Кто-нибудь шарит за Constraint Programming? Это случаем не очередная версия пролога?
Аноним 02/05/22 Пнд 21:03:06 95591 72
>>95590
>Programming
по ебалу давно не получал?
Аноним 02/05/22 Пнд 21:55:29 95595 73
>>95590
Похоже на декартово замкнутые категории с монадами.
Аноним 03/05/22 Втр 01:14:15 95602 74
>>95591
Чем тебе не нравится вопрос про прикладную матешу в общем треде /math ? Или предлагаешь с мат. оптами идти на /pr ?
Аноним 03/05/22 Втр 02:15:13 95605 75
>>95602
>Чем тебе не нравится вопрос про прикладную матешу в общем треде /math ?
у вас уже был свой тараканий тред

>предлагаешь с мат. оптами идти на /pr ?
лишь бы отсюда
Аноним 06/05/22 Птн 12:11:44 95683 76
Одна из мотиваций дифформ это тот факт, что функции нельзя интегрировать на произвольном многообразии (по крайней мере без доп. структур). Поскольку выделенной координатной системы нет, то величина интеграла от одной и той же функции изменится от простой смены координат/атласа (напр. $\int_0^1 dx$ перейдет в $\int_0^2 dx$).

С формами знаком уже давно поскольку они практически везде возникают (даже в алгеме, к моему удивлению).
Но вот пришлось мне теперь их преподавать и я не могу строго ответить на следующий (возможно, не корректно поставленный) вопрос одного из студентов:

Как именно, на "фундаментальном" уровне, формы решают эту проблему? В евклидовом пространстве пример интеграла выше разрешается очень просто - в евклидовом пр-ве есть доп. структуры (в частности, выделенная система координат), поэтому любая функция по дефолту это функция в канонических координатах, и тогда по известной теореме о якобиане\замене переменных мы получаем значение интеграла в других координатах.
Как можно, в этом же контексте, объяснить, как формы решают эту проблему для произвольного многообразия? Из определения интегрирования по цепи кажется, что всё равно в какой-то момент (при пуллбэке?) "индуцируется" значение в канонических координатах.

Не знаю, как лучше объяснить вопрос, может кто-то тут поймёт, что я имею ввиду.
Аноним 06/05/22 Птн 14:10:06 95684 77
>>95683
я не понимаю вопрос
дифф. форма есть инвариантый объект, не зависящий от системы координат. потому и интеграл от неё корректно определён (правда, должна быть выбрана ориентация)
или тебе нужно бескоординатное определение дифф. формы и интеграла от неё? я бы поискал в записках лекций вербицкого, наверняка у него что-то есть
не спаривание же в когомологиях для этого вводить

>Одна из мотиваций дифформ это тот факт, что функции нельзя интегрировать на произвольном многообразии
я с этим, кстати, не совсем согласен
функции вообще нельзя интегрировать
под "доп. структурой" здесь можно понимать только выделенную форму объёма, т.е. всё равно интегрировать будем форму (функция помноженная на форму объёма).
Аноним 06/05/22 Птн 14:56:58 95685 78
>>95684
>функции вообще нельзя интегрировать
Да я именно это и имел ввиду, в функциях зашито недостаточно информации и они вообще не тот объект, который можно интегрировать.
>потому и интеграл от неё корректно определён
>инвариантый объект, не зависящий от системы координат.
Но в итоге мы всё равно домножаем на якобиан при пуллбэке, то есть масштабируем - и мой вопрос это "масштабирование по отношении к чему?"
В евклидовом случае с формой объёма всё понятно, но и при интегрировании форм на проивзольном многообразии мы всё равно должны знать про масштаб. В какой-нибудь $\omega=a(x)dx$ коэффициент $a(x)$ в итоге отвечает за величину интеграла, я про это и спрашиваю.

То есть мы всё равно в итоге аппелируем к форме объёма евклидова пространства, потому что интеграл по цепи произвольного многообразия определяется через пулбэк как обычный интеграл от функции.
Аноним 06/05/22 Птн 23:17:47 95693 79
>>95685
Влезу немного с оффтопом, однако функции это есть 0-формы и их "интегрировать" можно.
Аноним 07/05/22 Суб 07:47:55 95695 80
>>95693
>Влезу немного с оффтопом
>однако функции это есть 0-формы
хоть если влезаешь и хочешь сумничать на технических деталях, не говори неправильной хуйни
Аноним 07/05/22 Суб 10:19:55 95700 81
>>95693
при такой интерпретации их придётся интегрировать по 0-мерному многообразию, т.е. по точке

>>95685
умножение на якобиан это просто закон изменения дифф. формы при замене координат, про никакое "масштабирование" мне непонятно.

мы всё же, наверно, не совсем правы, что функции нельзя интегрировать, их интегрировать можно, для этого нужна мера. форма объёма на ориентированном многообразии определяет меру, отсюда получаем интеграл

всё же я бы всё равно рассуждал, что интеграл от формы на ориентированном многообразии корректно определён, т.к. правильно ведёт себя при замене координат, а в координатах это интегрирование на $\mathbb R^n$, т.е. предел интегральных сумм; хотя такое рассуждение, наверно, следует считать не очень приличным, но именно так я себе всё это представляю. в сущности, "форма объёма определяет меру" всё равно сводится к этой конструкции.

>мы всё равно в итоге аппелируем к форме объёма евклидова пространства
если говорить про многообразия, то это на самом деле не совсем правильно, т.к. стандартная форма объёма на $\mathbb R^n$, которое получается из карты, зависит от выбора координат. т.е. мы к ней не аппелируем, её, собственно, на многообразии нет
Аноним 18/05/22 Срд 09:59:29 95881 82
в столбик.jpg 207Кб, 1280x720
1280x720
Как называется эта хуйня?
Аноним 18/05/22 Срд 12:00:00 95883 83
>>95881
поправка на 2-коцикл
Аноним 18/05/22 Срд 13:48:39 95885 84
>>95883
Чет пучкнул с твоего поста.
Аноним 18/05/22 Срд 18:02:49 95887 85
>>95881
У тебя сложение по модулю 10. Вот и думай.
Аноним 18/05/22 Срд 18:32:58 95888 86
>>95887
Я школьник — хули я там "надумаю"?
Аноним 18/05/22 Срд 21:40:56 95889 87
>>95888
>Я школьник
Ну так смотри что такое позиционная система счисления
Аноним 18/05/22 Срд 21:56:26 95890 88
Можете посоветовать книжек по функциональному анализу?
Аноним 18/05/22 Срд 23:01:14 95891 89
>>95890
Хелемский - Лекции по функциональному анализу
Аноним 19/05/22 Чтв 12:53:05 95894 90
Аноним 19/05/22 Чтв 19:46:21 95904 91
>>95894
Автор пытается обучить студента с категорных позиций. Для молодого студня на матфаке такое изложение может оказаться полезным. Впрочем полезно параллельно взять какой-нибудь классический учебник по функану, того же Секефальви-Надя
Аноним 19/05/22 Чтв 22:27:27 95906 92
>>95904
Колмогоров, Фомин - в топку?
Аноним 20/05/22 Птн 00:32:04 95909 93
>>95906
Для первого знакомства норм

Мне лично саймон-рид больше других нравится,
хотя любимого учебника по функ.ану у меня нет

На самом деле в этой науке много специфических вещей, которые могут толком никогда и не понадобиться, так что поверхностного знакомства (колмогоров-фомин) может хватить. Наверное

мимо
Аноним 20/05/22 Птн 10:25:57 95915 94
>>95906
Ну кстати тоже вполне может подойти. Проблема тут в том, что функан вещь крайне, крайне широкая.
Аноним 20/05/22 Птн 14:02:09 95916 95
>>95909
>Мне лично саймон-рид больше других нравится
Двачую, но советовать не стал, потому что это, все же, функан для (мат)физиков.
Аноним 24/05/22 Втр 17:59:47 96006 96
Как правильно читать Бурбаков? Чему уделить внимание, а что лучше скипнуть?
Аноним 24/05/22 Втр 20:09:35 96010 97
>>96006
Смотря с какой целью читаешь. Если нужна теория множеств, то предварительно нужно прочитать разъяснение смысла тау-символа в книжке Гильберта (только у него эпсилон, а не тау). Если коммутативка - том про множества можно не читать.
Аноним 25/05/22 Срд 05:41:35 96023 98
>>96010
Основная цель, как ни странно, познакомиться с трудами. Я слышал много хвалебных слов (ровно как и ненависти) в сторону Бурбаки, но сам под влиянием школы Арнольда никогда их не читал.
Аноним 25/05/22 Срд 07:28:43 96024 99
>>96006
Алгебру следует читать в английском переводе, так как у русни старое издание, в котором нет точных последовательностей.
Аноним 25/05/22 Срд 18:34:02 96030 100
Любой марковский объект может быть представлен в виде марковской матрицы, не так ли? И цепь, и сеть, и любые рандом волки?

Если так, то почему постоянно употребляются излишние термины по типу машины Больцмана/Хопфилда/Байеса/волков в примате? Если не сводятся, то, видимо, я чего-то не понял, а, если сводятся, то почему это не генерализировать в простой и понятный школьнику объект в виде матрицы?
Аноним 10/06/22 Птн 10:36:08 96277 101
Аноним 11/06/22 Суб 16:13:34 96286 102
Антоши, весь день не могу найти, может вы поможете.

Как называется эффект, когда человек сосредоточен на цифрах и неосознанно потом все делает в угоду цифр ?

Классический пример, когда у сотрудника стоит KPI, он начинает не работу работать, а цифры трахать.
Аноним 11/06/22 Суб 16:39:08 96287 103
>>96286
В приведённом тобой примере это называется здравый смысл. Если сотрудник будет делать что-то полезное вместо KPI, то его взъебут. Мне кажется, ты не совсем догоняешь, как оно во взрослом мире работает.
Аноним 11/06/22 Суб 17:06:38 96289 104
>>96287
Мне кажется ты совсем не догоняешь мой вопрос и пытаешься тут блеснуть умом, но ты облажался - завязывай учить людей взрослому миру.

Я не спрашиваю как руководить, я спрашиваю как называется ЭФФЕКТ.
Аноним 11/06/22 Суб 17:13:46 96290 105
>>96289
Я не знаю о чём ты спрашиваешь, но в приведённом тобой примере поведение сотрудника строго рациональное.
Аноним 11/06/22 Суб 17:17:43 96292 106
>>96290
Забей, найду, скину…
Ищу точное название эффекта аналитических искажений.
Аноним 11/06/22 Суб 18:14:17 96295 107
Гайс, можно ли повысить интерес к математике?? Например, сейчас учу по алгебре пространства (преобразование, приведение, евклидово пространство и тд), но как же скучно читать теорию и доказательства. Что это? Нехватка воображения?
Аноним 12/06/22 Вск 14:52:09 96304 108
Аноним 12/06/22 Вск 15:13:01 96305 109
Аноним 12/06/22 Вск 16:55:42 96309 110
Реальная жизнь.png 18Кб, 400x535
400x535
Аноним 12/06/22 Вск 17:50:22 96313 111
image.png 707Кб, 612x612
612x612
image.png 447Кб, 720x630
720x630
>>96295
Тебе не нужен интерес, тебе нужен характер (с) Конфуций.
Аноним 14/06/22 Втр 12:57:27 96361 112
теорема о неполноте твоей мамки по сути о том, что к любой системе аксиом всегда можно добавить новых ещё и ещё (правда, всё более изъёбистых, так что и риск напороться на противоречие возрастает)?
Аноним 15/06/22 Срд 09:36:49 96370 113
Репостну вопрос, на который никто не ответил в треде для новичков.

Почему функториальность гомологий - это именно то свойство, которое позволяет считать гомологии из локальной информации? Не понимаю связи.
Аноним 10/08/22 Срд 13:10:44 97688 114
>>10487 (OP)
Программа гуманитарного дебила-второкурсника для скатывания в математику (требует критики, если анону не сложно).

Задача - вкатиться в тервер и статистику, перейти от решения задач по алгоритмам из шараги к построению, выводу и доказательству теорем.

Г О Д 1
1. Richard Hammack –Book of Proof - научиться основам построения математического доказательства, чтобы не пригорать жопой от задач на доказательство. (https://www.people.vcu.edu/~rhammack/BookOfProof/)
2. Justin Hill et al. Elementary Abstract Algebra: Examples and Applications - разобраться с основными понятиями в математики отображение, функция, кольцо, группа и т.д. (https://open.umn.edu/opentextbooks/textbooks/850)
3. Michael Spivak Calculus - тупо разобраться с основами анализа, обмазаться пределами и дифференциальным исчислением.
4. Sheldon M. Ross A First Course in Probability - вкатиться в тервер, урча матаном. (http://julio.staff.ipb.ac.id/files/2015/02/Ross_8th_ed_English.pdf)
5. Wackerly, Mendenhall, Sheafer Mathematical Statistics with Applications - добить бэкграунд в статистике до рабочего уровня, чтобы мочь пойти стажером в пару интересных мне контор.

Г О Д 2, Г О Д 3 (книги российских авторов, но читать буду на английском, ясное дело)
1. Винберг Курс алгебры - повторить алгебру, уже строже
2. Зорич Математический анализ - повторить матан, уже глубже
3. Ширяев Вероятность - 1, Вероятность - 2 - вкатиться в тервер с нормальной базой, получить задел за пределы чисто прикладных задачек
4. T.W. Anderson An Introduction to Multivariate Statistical Analysis - развиваться в сторону освоения методик многопеременного анализа и т.д., что пригодится в магистратуре.
Аноним 10/08/22 Срд 15:25:01 97689 115
совет.png 55Кб, 735x723
735x723
>>97688
Хотелки понятны. Как реализовывать-то собираешься? Надеешься, что оно "само поймется и навсегда запомнится"? Лол, нет.

Британская мета.
Kevin Houston, "How to think like a mathematician".
Lara Alcock - How to Study for a Mathematics Degree (2012, Oxford University Press)

Максимально мягкий британский вкат.
Lara Alcock - How to Think About Abstract Algebra (2021, Oxford University Press)
Lara Alcock - How to Think About Analysis (2014, Oxford University Press)

Полноценный вкат в пруфинг. Оче популярен на матховерфлоу.
Gary Chartrand, Albert D. Polimeni, Ping Zhang, "Mathematical Proofs. A Transition to Advanced Mathematics".
John P. D'Angelo, Douglas B. West, "Mathematical Thinking: Problem-Solving and Proof".

Вкат в наивную теорию множеств.
Верещагин, Шень, "Начала теории множеств".
Видеолекции С. Сперанского, "Математическая логика и культура математических рассуждений".

Советы вообще.
Во-первых, задачи важнее теории. Треть времени на теорию - две трети времени на задачи. Хуйня без задач вроде Винберга в эту схему не вписывается - так что либо меняй, либо дополняй задачником.
Во-вторых, учебники, особенно салфетские, содержат очень мало мотивационной части. Автодидакты особенно часто дают заднюю именно по этой причине. Поэтому на каждый учебник ищи 1-2 видеокурса, где возле доски будет бегать живой человечек, махать руками, отвечать на вопросы аудитории и ОБЪЯСНЯТЬ материал. ОБЪЯСНЕНИЕ на порядок важнее самого доказательства.
В-третьих, Ширяев это мем, и самостоятельно практически непроходим. Наверное, это второй по уровню троллинга мем после Зорича.
В-четвертых, не имеет значения, чё ты там выучил "вообще". Никогда не учись ничему "вообще". Общность вторична, конкретика первична. Твоя конечная цель - проскочить через фильтр отбора в виде экзамена или собеса. Поэтому ходи с черным мешком и собирай конкретные примеры вступительных и собесов. Будешь выебываться и гениальничать - соснешь хуйца.
Аноним 10/08/22 Срд 19:14:44 97691 116
>>97688
>>97689
Зачем читать книги о "как доказывать" когда можно доказывать?
Возьми ЛМК, а затем Алфутову. Параллельно можно без особого напряга читать, например, Числа и Фигуры Радемахера.
Аноним 11/08/22 Чтв 04:24:12 97697 117
16428496169720.png 480Кб, 482x622
482x622
>>97691
К ЛМК и Алфутовой должен прилагаться заслуженный учитель россии и кружковский актив - потому что коллекция задач это одно, а умение их решать это совсем другое. Плюс выборка задач нерепрезентативна - олимпиадный душок, эвристически ориентированная дидактика, вот это вот всё. Если ты не двенадцатилетний школьник, а здоровый лоб с нормально созревшим для абстрактного мышления мозгом, то начинать нужно с начал - логика, наивная теория множеств, элементарные структуры, элементарные методы доказательств. Потом уже можно отрабатывать навыки на сборниках кружковских задач и ковыряться в олимпиадных эвристиках.

Для старшеклассников, студентов и взрослых автодидактов ничего лучше западной транзитной литературы так до сих пор и не придумано. Да и быть не может. Ну, разве что репетитор и личный олимпиадный коуч - но такое по карману не только лишь всем.
Аноним 11/08/22 Чтв 07:24:21 97698 118
>>97697
>Потом уже можно отрабатывать навыки на сборниках кружковских задач и ковыряться в олимпиадных эвристиках.
Зачем, если это можно делать сразу же?
Аноним 11/08/22 Чтв 13:33:17 97703 119
>>97689
>Надеешься, что оно "само поймется и навсегда запомнится"? Лол, нет.
Решать задачки на досуге? Все равно по дороге в метро нечего стало делать, как отключил мобильный интернет, чтобы не залипать на новости и политоту.

>Полноценный вкат в пруфинг. Оче популярен на матховерфлоу.
Спасибо, посмотрю!

>Вкат в наивную теорию множеств.
А почему наивная теория множеств, а не аксиоматическая?

>Во-вторых, учебники, особенно салфетские, содержат очень мало мотивационной части. Автодидакты особенно часто дают заднюю именно по этой причине.

Я надеюсь восполнить этот пробел тупо решением интересных мне задач или же удовольствием от понимания того, что раньше делал как по рецептуре из поваренной книги, на более абстрактом уровне.

>В-третьих, Ширяев это мем, и самостоятельно практически непроходим. Наверное, это второй по уровню троллинга мем после Зорича.
Разве? Я полистал - мне показалось вполне проходимым вместе с задачником от этого же автора, если усвоить базовые навыки доказательства.

>В-четвертых, не имеет значения, чё ты там выучил "вообще". Никогда не учись ничему "вообще". Общность вторична, конкретика первична.

Ну я знаю для чего я что-то учу. Но как и в случае иностранных языков, чтобы хорошо говорить (а не выдавать "моя твоя понимайт") - надо потратить время на изучение как алфавита, так и морфологии и синтаксиса. Но как только доберусь до хотя бы матанализа и интегрального исчисления - дальше понятно, что делать.

Спасибо большое за советы по книгам - все посмотрю и, возможно, поменяю программу чтобы вкатиться с большей вероятностью!

>>97691
А что такое ЛМК? Ленинградские Математические Кружки?
Аноним 11/08/22 Чтв 15:01:12 97705 120
image.png 800Кб, 1024x1024
1024x1024
>>10487 (OP)
Н.О.Д
Почему? Что они скрывают? Зачем они назвали себя в честь наибольшего общего делителя?
Натуральные числа Zа русских??
Аноним 22/08/22 Пнд 12:31:33 98119 121
>>10487 (OP)
Привет всем, может кто подскажет сколько часов нужно чтобы разобраться с матанализом, линейной алгеброй, статистикой и теорвером? Просто чтобы было понимание, в доктора наук не мечу.
Старт можно сказать нулевой, школа хоть и физмат профиль
И заодно может есть рекомендации? У меня уже список литературы нормальный собрался, но может чего и добавлю себе
Аноним 22/08/22 Пнд 15:48:37 98126 122
Аноним 22/08/22 Пнд 22:58:08 98151 123
>>98126
9 месяцев это из расчета сколько часов в день? 6-8?
Аноним 23/08/22 Втр 08:10:05 98155 124
>>98151
По каждому предмету часов 3-6 в неделю
Аноним 23/08/22 Втр 10:55:26 98160 125
>>98155
О, отлично, я могу заниматься даже больше! Спасибо :)
Аноним 29/08/22 Пнд 16:48:30 98420 126
Есть ли в математике сейчас что-то более модное, чем алгебраическая геометрия, которой уже полвека?
Аноним 02/09/22 Птн 02:07:07 98605 127
Математика - это такой чмошник-омеган среди наук, про которого вспоминают только когда имеющийся матаппарат не справляется с новыми задачами описания мироздания естественными науками, когда этот чмошник выполняет свою функцию про него благополучно забывают.
Любой ученый-специалист какой-либо из естественных наук в совершенстве владеет математикой, но ни один математик нихуя не понимает ни в физике, ни в химии, ни в биологии на достаточном уровне.
Более того, опыт и навыки полученные математиком при изучении своей науки никак не помогут ему в естественных науках, ибо они оторваны от реальности и не ставят своей целью познать ту самую реальность, напротив, они занимаются логическим онанизмом и ментальной гимнастикой.
Математика - это страх, боль, деградация, депрессия и упадок в то время как естественные науки - это здоровье, счастье, гармония, процветание и просветление.
Аноним 02/09/22 Птн 02:14:21 98606 128
>>98119
Матанализ - 2 семестра по 3 месяца.
Линейная алгебра - 1 семестр
Статистика - 1 или 2 семестра, не помню
Теорвер входит в статистику и дискретную математику, на нее тож семестр нужен
И того полгода. Ток плотно надо сидеть, так как ты школотрон растягивай на год, как и анон выше сказал.
Если ты в доктора наук не метишь нахуя оно тебе? Алсо нахуя оно тебе в любом случае, в любом профильном вузе тебя задрочат им энивей а если нет то и нахуй оно тебе не надо.
Мотивация "ХАЧУ ЗНАТЬ" достаточно слабая и уже на третьей главе ты пойдешь аниме смотреть. Если только ты этим не интересуешься на уровне хобби, тогда чего ты ждал, короч непонятно.
Аноним 02/09/22 Птн 04:30:51 98607 129
>>98605
>Любой ученый-специалист какой-либо из естественных наук в совершенстве владеет математикой
Лол. Сколько из них хотя бы знает определение пучка или расслоения?
Аноним 02/09/22 Птн 09:26:24 98611 130
>>98605
Толсто, переделывай
Аноним 06/09/22 Втр 18:25:52 98733 131
аноны
допустим есть 10 цифр от 0 до 9
какое количество чисел длиной 5 я могу составить?
я посчитал, у меня получилось 30240
делал комбинаторикой.
на первом месте может быть любая из 10 цифр, на втором из 9, и т.д
в чем я не прав?
Аноним 06/09/22 Втр 18:26:40 98734 132
>>98733
это при условии того, что цифры не могут повторяться
Аноним 06/09/22 Втр 18:28:31 98735 133
>>98734
а если бы цифры могли повторяться?
поясните
в школе учусь, эту тему только проходим. прошу не бить ссаными тряпками
Аноним 13/09/22 Втр 12:17:14 98875 134
Искривление пространства и времени, это ведь на деле искривление отношений внутри координат пространства и времени, а не их самих. Ведь изменение пространства и изменения внутри пространства - это разные вещи. Но этот различие, есть чисто умозрительное, а не реальное различие. Если исходить из того, что пространство и время, есть лишь часть нашей субъективной интуитивной деятельности, которая позиционирует наши разрозненные ощущения в опыт, и именно поэтому же даёт возможность "усматривать" отношения внутри себя. То мы тем самым получаем внешнее содержательные объективные отношения и внутреннее идеальное пространство позиционирующие их. С одной стороны у нас сохраняется необходимость Математики, с другой у нас Математическое пространство не является "объективным". Точнее, нет вообще никакого пространства кроме нашего субъективного, которое есть лишь обработка наших ощущений, как некая эволюционная надстройка. Правда в таком случае, рано или поздно, умозрительное пространство и время, должно столкнутся в опыте с необходимостью перейти на приборное объективное отношение, которое должно расходится с пространственно-временным корнем нашего восприятия. Крч, эксперементы и матмодели скоро закончатся и всё математики потеряют работу. Учите питон и джава скрипт, иначе умрёте с голоду. что думаете вы о Пространстве и времени?
Аноним 13/09/22 Втр 12:40:03 98876 135
>>98875
научпоп шиза в >>>/sci/
сюда с вопросами по собственно математике, в данном случае по дифференциальной геометрии
Аноним 19/09/22 Пнд 19:24:42 98991 136
Есть в игре головоломка, надо типа на весы разложить 3 типа монет с разным весом и получить вес 107 грамм. Способом рандомных нажатий я значение получил, но хочу это по умному решить. Дайте подсказку, как это можно решить.
Аноним 19/09/22 Пнд 21:17:24 98996 137
>>98991
Ладно, сам решил.
Аноним 21/09/22 Срд 00:28:27 99046 138
>>98875
Начни с дифференциальной геометрии, прям с определения метрического пространства и метрического тензора.
Аноним 22/09/22 Чтв 13:13:52 99059 139
Можете объяснить, как по какой формуле считать количество комбинаций как в вот этой задаче >>98991?
Если есть всего 3 ячейки в каждой из них может быть число от 0 до 10? Я написал программу, которая насчитала 1332 комбинации, но интересно формулу узнать. Я загуглил, там что-то про основания, мне это непонятно.
Аноним 24/09/22 Суб 13:34:20 99086 140
1 Аноним 22/10/22 Суб 16:35:57 99462 141
Нахуй оно нужно, объясните? Это простое число убивает всю математику на корню.
Аноним 23/10/22 Вск 09:59:04 99473 142
Аноним 26/10/22 Срд 17:44:30 99520 143
>>99462
Как по мне, самое пиздецовое число 2 потому что такой пиздец происходит когда ты сидишь в char 2.
Аноним 21/11/22 Пнд 07:13:52 99909 144
>>10487 (OP)
Либо я хуево искал, либо не нашел
Как научиться решать задачи? Вот у меня есть, например, демидович - как мне его решать? В нем даже примеров задач нет, только краткая справка, которой недостаточно.
Иными словами - как самому прорешать демидовича?
Аноним 22/11/22 Втр 08:08:37 99931 145
Аноним 22/11/22 Втр 09:34:05 99932 146
>>99931
а схуев он должен сходиться?
Распиши условия сходимости и увидишь
Аноним 22/11/22 Втр 12:17:24 99939 147
>>99932
Необходимое условие очевидно выполняется, потому что простые числа не заканчиваются.
А достаточные я хуй знает как тут применять, вот и спрашиваю.
Аноним 23/11/22 Срд 01:28:01 99944 148
>>99939
Асимптотика простых чисел известна, соответствующий интеграл расходится.
Аноним 03/12/22 Суб 11:33:31 100094 149
Кокая зорплата у топовых математиков, пашущих на топовых кабанчиков в уол-стрите (или также являющихся таковыми)? Или там скорее кодеры-нейросеки бабос гребут?
Аноним 03/12/22 Суб 12:57:53 100096 150
45494.png 114Кб, 858x913
858x913
Аноним 06/12/22 Втр 15:50:24 100128 151
61121532613.png 163Кб, 1338x696
1338x696
7602011092855.png 207Кб, 1218x965
1218x965
Читаю факультативный курс по алгебре для всех желающих. Щедро сдабриваю повествование всякими примерами и картиночками а-ля пикрелейтед. Мне один кфмн с кафедры говорит, мол, ты отучаешь их мыслить своими картиночками, если каждое базовое понятие вроде факторгруппы будешь разжёвывать. Но читаю я не чистым математикам (и даже не математикам вовсе). Как считаете, он прав?
Аноним 07/12/22 Срд 09:15:11 100133 152
>>100128
>Как считаете, он прав?
Нет. Мой утилитарный аргумент заключается в том, что ты экономишь время студенту.
Также, ты показываешь что можно использовать мнемотические схемы для укрепления учебного материала, если твои рисунки не пригодятся, то они сделают свои. Если даже студенту это не потребуется. То ничего с него не будет.
Аноним 07/12/22 Срд 14:41:45 100135 153
image.png 606Кб, 606x714
606x714
>>100128
>ты отучаешь их мыслить своими картиночками
Запахло самым вкусным пломбиром и фихтенгольцем.

Рикамендую попросить кфмн для начала дать определение понятию "мыслить". Как правило, наглядные сравнения, иконографические помавания руками и попытки что-то накарябать на случайном листочке возникают уже на второй-третьей итерации уточнения исходного определения. Решение задач - любых задач - всегда сопровождается имаджинированием и мышечной моторикой, это экспериментально установленный факт.

Манипуляция символами изначально возникла как абстракция от механического манипулирования объектами в поле зрения. Знаки это те же вещи. От натуральных объектов они отличаются только меньшим количеством признаков, подобием форм и универсальностью конструкции. Буквы в этом посте утратили все признаки, кроме ширины, высоты и толщины линии, число цветов сократилось до одного; все они похожи друг на друга цветом, размером и углом наклона; все они сконструированы из универсального конструктора (отрезков и дуг) по универсальным правилам - поэтому мы сразу распознаем эти маленькие черные объекты в качестве знаков. Даже если бы этот пост был написан на высоком валирийском, мы бы все равно поняли, что имеем дело с какими-то объектами-знаками.
Аноним 09/12/22 Птн 16:45:26 100178 154
>>100128
Я бы ахуел с таких картинок. В них тяжелее разобраться, чем в буковках, как мне кажется.
Раз уж нематематикам рассказываешь, то думаю лучше рассказывать им в том видео, в каком это появилось.
Можешь, например, поставить цель - решить уравнение 4 степени. Не трюками, типа метода Феррари, а проанализировав решение кубического уравнения. Там каждый радикал в формуле Кардано - это какая-то функция от корней, причём каждое значение радикала можно получить из этой же функции просто переставляя аргументы. Каждой функции ставишь в соответствии группу перестановок, что оставляют её на месте. Легко заметить, что на фоне, чем ближе мы к решению, тем меньше становятся группы. Потому можно зайти со стороны групп к этой задаче, доказать, что S4 разрешима и найти её разложение, а дальше по каждой группе построить функцию от корней, относящуюся к ней.
Для студентов такой переход от школьной алгебры к группам и полям будет естественным и безболезненным.
Аноним 10/12/22 Суб 22:05:38 100195 155
>>99086
Авангард науки как всегда на дваче.
Аноним 10/12/22 Суб 23:43:33 100198 156
>>100128
Ну честно, я бы не понял...по крайней мере очень долго шел бы к пониманию теоремы Коши (в теории групп) и теорем Силова без КАРТИНАЧЕК. Они очень облегчили мне понимание, потом я конечно с полученной интуицией изучил алгебраические доказательства. Где где а в теории групп побольше нужно визуала.
Аноним 14/12/22 Срд 21:46:45 100261 157
У меня 40к кубков в бравл старс
16/12/22 Птн 13:32:50 100291 158
>>96361
О том, что существующая и используемая сейчас формальная логика это хуйня неработающая
Аноним 17/12/22 Суб 12:41:23 100306 159
Аноним 18/12/22 Вск 15:38:04 100318 160
Аноним 28/12/22 Срд 23:58:00 100496 161
котаны, хочу пройти школьный курс математики начиная с первого класса.
Я тупой, так что желательно много задачников.
Подскажите годные книги/сайты, пожалуйста.
Аноним 29/12/22 Чтв 14:43:03 100516 162
>>100496
Сканави с ответами в качестве задачника по алгебре. Гордина по выжимкам теорем геометрии - "Это должне знать каждый мат. школьник" и его же задачник: "Решение задачи 16".
Аноним 01/01/23 Вск 12:56:12 100594 163
image.png 251Кб, 1080x911
1080x911
>>10487 (OP)
задача с фочана

как решить?

Аноны говорят, что нижний правый треугольник равнобедренный и его углы 90, 45 и 45. Откуда они это берут?
Аноним 01/01/23 Вск 13:38:35 100596 164
>>100594
Видимо увидя подпись square, просто всё углы четырёхугольника подписали 90°.
Аноним 01/01/23 Вск 22:29:22 100608 165
image.png 153Кб, 934x809
934x809
>>100594
Я красивого решения не увидел. Находим углы, которые можем. С помощью теоремы синусов выражаем стороны внутреннего треугольника через сторону квадрата. По той же теореме выражаем неизвестный угол через найденные стороны треугольника. Сторона квадрата сокращается, получается некрасивая формула и некрасивый угол. Попробовал построить, вроде совпадает.
Аноним 01/01/23 Вск 22:32:41 100609 166
>>100608
По теореме косинусов
Аноним 04/01/23 Срд 13:33:33 100629 167
16726013632380.png 191Кб, 934x809
934x809
>>100594
>>100608
зацените, как вам? Мне кажется, решилось довольно просто и без ошибок
Аноним 04/01/23 Срд 13:36:20 100630 168
>>100629
Я на картинке неправильно указал угол Альфа. Альфа это искомый угол по условию задачи. А я указал, так сказать, угол из промежуточных вычислений...
Аноним 04/01/23 Срд 15:27:47 100631 169
12321.png 164Кб, 934x809
934x809
>>100608
>>100594
Ошибся в первом решении в мелкой детали, сейчас 100% правильно.
Аноним 16/01/23 Пнд 02:13:37 100791 170
Pd8j7GYbfTE.jpg 93Кб, 750x674
750x674
Математик хуже пидораса

пруф ми вронг
Аноним 20/01/23 Птн 17:33:28 100875 171
>>100791
Неправда, математик лучше пидараса.
Аноним 30/01/23 Пнд 12:49:58 101024 172
Почему теория чисел со всеми этими вашими шифрованиями строится на элл кривых - т.е. уравнениях 3 степени, а кривые 4+ степени не рассматривают? Просто слишком сложно или какой-то глобальный смысол в числе три есть?
Аноним 30/01/23 Пнд 14:50:01 101026 173
>>101024
ну чтобы что-то шифровать нам нужно, чтобы была структура группы, но мы еще работаем с уравнениями, а они задают нам многообразия, если потребовать и структуру группы и многообразия, то такие штуки называются абелевы многообразия. ну и так получилось, что абелевы многообразия размерности 1 (кривые) получаются только из якобианов кривых рода 1, а кривые рода 1 задаются уравнениями 3 степени. Т.е. только для эллиптических кривых верно что само многообразие совпадает со своим якобианом, как множество якобиан будет группой пикара, ну а группа пикара -- это ... группа, поэтому на эллиптических кривых и есть структура группы. Ну а на уравнениях высших степеней так просто криптографию не построить, там на кривых уже нет структуры группы, нужно брать ее якобиан, а он уже не будет совпадать с самой кривой и будет иметь размерность больше единицы. По уравнениям высших степеней, например по кривым рода 2 можно строить абелевы поверхности с помощью их якобианов, и это конечно будет сложнее, но пока людям достаточно непробиваемости обычных эллиптических кривых
Аноним 31/01/23 Втр 13:18:28 101031 174
>>101026
написал кучу всего бесполезного, но не объяснил, почему собственно род 1
группа+многообразие -> канонический ПУЧОК тривиален -> по Риману-Роху род равен 1
или даже ещё проще, фундаментальная группа абелева -> род не больше 1
ну и пользы мало от этого, очевидно же что он залётный

>>101024
гугли hyperelliptic cryptography
в частности https://en.wikipedia.org/wiki/Hyperelliptic_curve_cryptography
пример статьи - гугли "Efficient Arithmetic on Genus 2 Hyperelliptic Curves over Finite
Fields via Explicit Formulae"
Ну понятно, что ты не математик, поэтому я тебе дам нормальное рукомахательное научпоп объяснение, а не как анон выше. Что такое в итоге этот твоё сложение на кривой? "Сложить" две точки A и B это провести через них прямую, и посмотреть, какую третью точку С она ещё пересечёт; тогда А+B=-C. Понятно, что нам тогда нужны такие кривые, где всегда можно провести прямую через три точки кривой. По теореме Безу (https://en.wikipedia.org/wiki/Bézout%27s_theorem), две проективные кривые степени m и n пересекаются в m x n точках с учётом кратности. Здесь степень - это как в школе, максимальная степень в уравнении кривой. У прямой степень = 1. Тогда у нашей кривой должна быть степень m так, что m x 1 = 3. Отсюда m=3, то есть это кубическая кривая.
Это конечно не значит, что нельзя определить сложение на других степенях, но там нужно будет что-то делать с "лишними" пересечениями, или вообще определять по-другому.
Аноним 01/02/23 Срд 17:46:16 101052 175
image.png 45Кб, 1730x838
1730x838
image.png 1738Кб, 1920x1080
1920x1080
Пытаюсь найти уравнение подвесного моста в minecraft. Использовал уравнение цепной линии (https://ru.wikipedia.org/wiki/Цепная_линия) и получается что-то в духе (для длины моста 56 и высоты 5):
f(x)=round(cosh((x)/82.0451)82.04512)/2-82
Дальше считал длину отрезков вручную, но интересно, как сделать это графиком. Сама длина отрезков стабильно уменьшается с высотой, но если попробовать округлить х, то она скачет, что выглядит не красиво и не естественно.

Нашёл хороший сайт для графиков: https://www.desmos.com/calculator?lang=ru
Аноним 01/02/23 Срд 17:48:16 101053 176
>>101052
f(x)=round(cosh((x)/82.0451)82.04512*2)/2-82
Двойка отклеилась
Аноним 01/02/23 Срд 18:39:58 101054 177
Я работаю на дноработе червепидором перекладывателем бумажек за 20к. Как думаете, за год при такой работе физически нетяжелой получится ли выучить весь матан, алгебру и геометрию, численные методы, теорию вероятностей и мат. статистику за джва курса технического ВУЗа?
Аноним 01/02/23 Срд 20:06:53 101055 178
>>101054
если кодером - нет, мозг вскипает за смену, заниматься после вредно и бесполезно
если 8-часовая рутина в офисе - нет, мозг от стресса не даст работать креативно над трудным материалом
офиком 2x2 по 12ч - уже лучше, no stress работа, два полных дня на полноценные занятия
самый норм варик - репетиторство и дно-работы на несколько часов(курьер, уборщик в духе good will hunting, реселл, завод - главное чтоб на пару часов, no stress, желательно физический труд)

Сможешь разгрузить голову от рабочих/бытовых/любых других стрессоров - сможешь регулярно, беззаботно и весело заниматься - the sky is the limit, все будет зависеть от твоей работоспособности, мотивации, реализации учебного планаИМХО.
Аноним 04/02/23 Суб 17:38:41 101099 179
>>101054
Если у тебя есть возможность заниматься прямо на работе. Или, как вариант, вставать часа в 3-4 утра и заниматься перед работой, со свежей головой.

>за год
Только на уровне зазубрить определения и методы решения типовых задачек + ориентироваться в справочниках.
И только если ты хорошо знаешь и помнишь школьный курс.
Аноним 16/02/23 Чтв 23:58:20 101325 180
image.png 98Кб, 800x1040
800x1040
>>10487 (OP)
Несчётность множества [0;1] везде предлагают доказывать через "диагональный аргумент", мол выпишем все числа в двоичной системе, возьмём цифры, обратные тем, что на диагонали, и получим число, которое не было записано. Но почему собственно полученное число должно оказаться новым? Почему обязательно не встретится такого числа раньше? Никто вообще не пытается это как-то обосновать.
Аноним 17/02/23 Птн 08:58:24 101341 181
>>101325
Можно и в десятичной записать, тогда на диагонали меняешь цифру на любую другую.
>Но почему собственно полученное число должно оказаться новым?
Потому что его нет в списке.
Допустим полученное число есть в списке. Будем сверять каждую строчку с нашим числом. Совпадает ли оно с 1-ым? Нет, первая цифра отличается. Совпадает ли с 2-ым? Нет, вторая цифра отличается и тд. Какое число из списка не возьми, в любом будет несовпадение. Значит такого числа просто нет в списке.
Аноним 17/02/23 Птн 09:03:54 101343 182
>>101341
Как пример можешь попробовать сдвинуть диагональ вправо и посмотреть что выйдет, или брать вертикальные/горизонтальные линии.
Аноним 25/02/23 Суб 23:13:28 101466 183
если бы ктото мог видеть в 4 измерениях график зетафункции, то он бы легко разгадал нипотезу римана?
Аноним 28/02/23 Втр 03:46:16 101522 184
Тест
[math]3/x=6[/math]

Мёртвая доска, двачеры слишком дебилы для такого.
Аноним 01/03/23 Срд 00:17:03 101539 185
1551471891758.png 66Кб, 524x276
524x276
1595096471644.png 74Кб, 520x276
520x276
1644637849952.png 42Кб, 521x275
521x275
1545810020444.png 255Кб, 670x1161
670x1161
Можно ли извлечь градиент с 2д картинки?
Допустим есть такие картинки. Понятно как они строятся. Но вот допустим я хочу по картинке (ну или таблице значений что то же самое) получить на выходе примерно, чем она нарисована. Радиус если радиальный, либо угол и длина если это линейный.
Помню что для обычных 1-мерных графиков есть какой-то способ, когда по значениям графика подгоняется формула, а тут?
Аноним 04/03/23 Суб 22:08:09 101611 186
Верно ли, что действительные числа более "реальны", чем мнимые, лишь потому, что мы живём в мире, где объекты чётко отделимы друг от друга, существует множество однотипных, поэтому у большинства людей в мозгах есть субитизация? А если б у большинства была дискалькулия или мы жили в мире перетекающих друг в друга туманностей, то натуральные числа были бы столь же абстракты, как всякие кольца-идеалы?
Аноним 05/03/23 Вск 17:20:19 101621 187
>>101611
Нет, неверно, в физике больше используются комплексные с мнимыми, а физика более чем реальна.
>объекты чётко отделимы друг от друга
А вот это как раз вымысел, поскольку деление на объекты как раз зачастую происходит в голове. Сколько волос составляет объект борода, перестройка корабля Тесея, дождь это объект или нет, вот это все.
Аноним 10/03/23 Птн 08:12:01 101696 188
Представляет ли отдельный интерес расширение поля алг чисел до множества корней многочленов, в которых и коэффициенты, и степени могут быть алгебраическими числами? А поле корней многочленов, в которых степени и коэффициенты - элементы этого поля,- это все комплексные числа?
Аноним 10/03/23 Птн 09:58:21 101697 189
>>101621
> Нет, неверно, в физике больше используются комплексные с мнимыми, а физика более чем реальна.
Реальна природа, а физика строит реалистичные модели. Она так же реальна по отношению к природе, как реалистичная игра с крутыми нпс - к миру и человечеству. Ну или по аналогии: вот раньше учёные изучали природу непосредственно чувствами: смотреть-слушать-щупать-нюхать; почти без инструментов, голыми руками. Теперь они ограждены от предмета перчатками, стенками приборов, камер, расстояниями, размерами. Примерно те же процессы произошли в уме: мы теперь объясняем природу не сущностями, образами и смыслами, а концепциями, не поддающимися образному описанию, для которых не придумали понятных слов. Наоборот: абстрактные слова вошли в повседневную жизнь (например, энергия).
> А вот это как раз вымысел, поскольку деление на объекты как раз зачастую происходит в голове. Сколько волос составляет объект борода, перестройка корабля Тесея, дождь это объект или нет, вот это все.
А голова сформирована эволюцией, которая дала человеку встроенные способы измерения: баесову вероятность, логарифмирование энергетических величин (яркость, громкость и тд) и субитизацию - мгновенный счёт до 4. Всё это у большинства людей одинаково. Есть дискалькулия, когда даже самая базовая арифметика дико абстрактна. А есть люди-калькуляторы, которые без труда видят в числах кучу дополнительных структур (типа разложения на простые) и связей друг с другом. Но большинство людей видят максимум величину, чётность, 0 и 5 на конце. Пи для них - просто 3.14, а не транцендентный пришелец из другой галактики.
Поле действительных чисел упорядоченно, комплексных - нет. У них два независимых параметра, неустранимая двухмерность. Видимо, здесь их сложность восприятия в сравнении с действительными. Ну или так: мы привыкли характеризовать числом некий итог, сведение воедино множества отношений. Всё в системе можно свести к чему-то одному и оценить этот результат. Это одна из самых базовых абстракций, у многих восприятие математики (и чисел) на этом и заканчивается. И вот оказывается, что комплексные числа для этого не подходят, число i "ничему не равно". Зато подходят для кучи всего другого. Например, для создания куда лучшего инструмента сведения системы к набору параметров - линейной алгебры.
Аноним 10/03/23 Птн 10:04:26 101698 190
>>101697
Так дискалькуляция - это просто обученная нейронка. Там же нет никакой математики. В каком-то советском журнале была такая же "распознавалка" цифр - канцелярские кнопки с проводами, прикладываешь 3 или 5 - замыкаются соответствюшие цепи и загорается ответ.
Аноним 10/03/23 Птн 10:29:51 101699 191
>>101697
>И вот оказывается, что комплексные числа для этого не подходят, число i "ничему не равно"
Ну если ты маняфилософ-пиздобол, в математике не разбирающийся от слова совсем - тогда конечно.
Аноним 10/03/23 Птн 13:11:24 101702 192
>>101699
Кавычки ты конечно не заметил но ты прав, в матанах я нихуя. Я имею в виду разрыв шаблона школьника, когда встречается с этим числом. С одной стороны, его приучили, что в выражениях типа 3а+5х вместо букв надо просто подставлять числа и получать одно действительное число,- но тут училка говорит, что 3+5i к одному действительному числу не сводимо (модуль означает лишь радиус окружности, а не точку на ней), и тут типа нечему удивляться. Типа ничё тут странного нет, тебе просто показалось. А на самом деле есть: древнейшая функция чисел - итоговая оценка, сведение множества к чему-то одному (и наоборот, т.е. простейшая биекция), однозначное сравнение с другими подобными и дальше по итерации. Если оценка хорошего исхода - число больше 0, то сумма нескольких исходов тоже всегда >0 в R. А в комплексных не так в жизни, впрочем, тоже не всегда, но
>>101698
хз о чём ты, дискалькулия - следствие мозгового дефекта
>В основе дискалькулии лежит отсутствие субитизации — способности оценивать количество объектов с первого взгляда (то есть без пересчёта). За эту функцию в мозге отвечает внутритеменная борозда теменной доли. У людей с дискалькулией данный участок мозга меньше, чем у большинства людей, и недостаточно активен.
Аноним 11/03/23 Суб 08:39:46 101713 193
>>101702
Ты крайне переоцениваешь ценность своих рассуждений. В /sci/ свою воду уже пробовал публиковать?
> С одной стороны, его приучили, что в выражениях типа 3а+5х вместо букв надо просто подставлять числа и получать одно действительное число,- но тут училка говорит, что 3+5i к одному действительному числу не сводимо
Школьников с ранних лет учат к гамильтоновской записи векторов как ai+bj+ck, где результат к действительному числу не сводим. Даже в простейшей аналогии ты обосрался, остальное даже и обсуждать не стоит. Открой какую-нибудь книгу для разнообразия, а не ютюб.
Аноним 11/03/23 Суб 14:22:23 101718 194
>>101713
> Школьников с ранних лет учат к гамильтоновской записи векторов
Но эти векторы никто не называет числами, сразу говорят, что них модуль и направление по нескольким осям, заставляют стрелочку сверху ставить, учат отдельно уравнения по физике в векторной и скалярной форме писать. Если б комплексные числа ввели как матрицы [a b,-b a], было б меньше лишней мистики.
> Ты крайне переоцениваешь ценность своих рассуждений.
Ну да, а потом недооцениваю. Способность к самокритике сохраняется, всё норм пока что.
> В /sci/ свою воду уже пробовал публиковать?
Пробовал. Только зачем сци, если есть отдельный раздел для матеши, а в нём - этот тред именно для воды. Твои претензии были бы понятны, если б я создал отдельный тред здесь (или на дхду, хотя всё ещё впереди) и кидал ссылку на него. А я во флудилке пишу. Камон, чувак, мы не в здании махмета, ты в публичном месте без фейсконтроля, сам сюда пришёл.
Аноним 11/03/23 Суб 14:30:12 101719 195
>>101718
>если есть отдельный раздел для матеши
Но ты то спрашиваешь про реальность.
А ответ тебе сразу был дан.
Аноним 11/03/23 Суб 14:50:18 101721 196
>>101719
>А ответ тебе сразу был дан.
Одна часть про реальность, я принимаю это. Другая про меня:
> Ты крайне переоцениваешь ценность своих рассуждений. В /sci/ свою воду уже пробовал публиковать?
Я ответил на это - душно, но спокойно. Что не так?
Ну а про реальность - я не говорил, что мнимые числа нереальны, я действительные реальны. Я про то, что они упорно воспринимаются такими потому я и поставил кавычки в самом первом вопросе. И я не нашёл, чтоб этот феномен кто-то разобрал до конца, шаг за шагом, везде фрагментарные объяснения. Например, мы хоть и знаем, что в древности отрицательные числа были столь же "нереальны", но не можем преставить, почему. Ну и комплексные числа исторически тоже из отрицательных возникли. Мне, во-первых, кажется, что в основе рациональных и комплексных чисел лежит абстрация несколько разных принципов, изначально противоречащих друг другу. Во-вторых, что расширение рациональных чисел до действительных - более абстракная операция, чем до комплексных.
Аноним 11/03/23 Суб 18:12:03 101722 197
16635465423723.png 497Кб, 640x512
640x512
>>101721
Все числа - как формальные дескрипции структур - одинаково нереальны. Субъективно воспринимаемая "степень реальности" структур зависит не от самих структур, а от моделей, через которые они могут быть представлены. Канонической репрезентацией натуральных чисел является конечное множество подобных друг другу объектов, например, камешков. "Нереальность" отрицательных чисел преодолевается введением естественной репрезентации целых чисел в виде дискретной оси координат - вперед от нуля плюс, назад минус. "Реальность" сводится к привычности.

Комплексные числа, как дескрипция, содержат в себе дескрипцию действительных чисел, как подстроку - а действительные точно так же содержат в себе рациональные. Собственно, в случае чисел, эволюцию структур можно представить в виде итеративного повышения complexity их дескрипций на некоторую дельту. Понятно, что величина этой дельты может отличаться от шага к шагу - между натуральными и целыми числами дельта меньше, чем между рациональными и действительными.

Операции расширения, представленные как некие алгоритмы работы со строками дескрипций, вообще говоря, могут очень сильно отличаться друг от друга. Расширение расширению рознь, надстраивать новые дескрипции над уже существующими можно тысячей различных способов. Одно дело пополнить рациональные числа с помощью операции предельного перехода - другое дело построить комплексные как ВП над действительными.

Короче, нужно различать как минимум два фактора - сложность самих структур и сложность алгоритмов их преобразования. Люди всегда тяготели к структурам и алгоритмам попроще - чем проще структура, тем легче подобрать ей "естественную" репрезентацию внутри создаваемого нашим мозгом трехмерного манямирка (то есть отобразить символы в камешки, линии на бумаги, плоскости и проч.), а чем "естественнее" репрезентация, тем она субъективно "реальнее". С операциями та же история - сложить две кучки камешков в одну гораздо "естественнее", чем проиллюстрировать предельный переход в виде вереницы геометрических фигур, исчерпывающих другую фигуру. После достижения некоторого потолка сложности, "естественность" структур полностью теряется - и начинается процесс размахивания руками, множественное представление фрагментов структуры в виде схем и рисунков и прочее колдунство - в надежде, что пациент сумеет нащупать в потоке частичных репрезентаций некоторый скрытый инвариант и понять подлинную суть происходящего.
Аноним 13/03/23 Пнд 11:09:49 101739 198
>>101718
Геометрическую интерпретацию комплексных чисел знали инженеры-самоучки столетия назад.
Сразу видно, что ты не математик, потому что как только речь заходит о матрицах, уже значит ты говоришь про сосны, не видя леса. Матрицы - это всегда представление каких-то объектов. И есть отличная геометрическая интерпретация умножения комплексных чисел, которая является первичной и собственно индуцирует твоё умножение матриц в подкольце матриц определённого вида.
Да и вообще это вопросы по преподаванию, к математике отношения не имеющие. Поэтому давай обратно в /sci/, /un/, /re/,/ psy/, и прочие графоманские доски.
>Только зачем сци, если есть отдельный раздел для матеши, а в нём - этот тред именно для воды.
Он "для воды" только в понимании второкуров-погромистов вроде тебя. Другие аноны задавали и задают тут конкретные вопросы по математике.
Аноним 14/03/23 Втр 00:50:31 101748 199
>>101739
>Да и вообще это вопросы по преподаванию, к математике отношения не имеющие. Поэтому давай обратно в /sci/, /un/, /re/,/ psy/, и прочие графоманские доски.
поддерживаю
полезли тараканы в который раз, понимаешь
ничего не знают, а языком помести им хочется
Аноним 18/03/23 Суб 08:41:31 101833 200
Реально ли вообще крепко выучить матан по 11 класс без препода?
То же и с русским. Очень сильно клал хуй на учебу в шкалке и пропустил кучу тем, образован на уровне 4 класса по сути, не знаю многих базовых терминов которые проходят в первых классах и они сидят в подкорке у каждого, а у меня не сидят и кажутся очень сложными для понимания.
Сам прошел пару тем по ютубу, вроде понял, но с практикой проеб, не знаю сколько их практиковать и где брать примеры.
Как я понял книги из шапки, даже самая первая, уже для тех кто имеет базу в 11 классов и хочет повторить или именно "понять" матан, а мне бы просто инструмент для вычислений и решений заадач иметь в голове
Аноним 30/03/23 Чтв 02:41:28 102052 201
Здравствуйте, я компьютерный мастер, недавно получивший заказ на починку компа. Пка старая, винда 2007. Анонче, помнишь, как мелким пиздюком залипал на виндовс проигрыватель, слушая тупую музыку? Так вот тут это тоже было. Я решил понастольгировать. Открыл проигрыватель. А там фракталы... Демонические образы... Непередаваемый необъяснимый ужас... Я застыл. Я трясся от страха. Я пошел спать и мне снились кошмары. Мне снилось множество
Мандельброта, треугольник Серпинского... Сотни пикселей тянулись ко мне, засасывали меня. Я был точкой, стремительно несущейся навстречу бесконечным структурам. Я больше не точка. Я -- фрактал. Это так естественно и правильно. Смотрю на руки. Ничего нет. Я не смотрю на руки, ведь меня нет. Моей комнаты нет. Дома. Улицы. Города. Нет ничего. Все сущее есть фрактал. Прекрасное Множество Мандельброта. Оно – все сущее. Чувствую, как раздувается, пухнет, ветвится и… Снова есть я. «Я» проснулся. Обнаружил что обосрался ночью.
Аноним 30/03/23 Чтв 22:26:09 102071 202
16601323951022.png 1079Кб, 1080x1350
1080x1350
Многоуважаемые матаны, спасити памахите! Нужна программа для доказательства тавтологичности формул состоящих из предикатов. Вот это вообще про это?
https://ru.wikipedia.org/wiki/%D0%97%D0%B0%D0%B4%D0%B0%D1%87%D0%B0_%D0%B2%D1%8B%D0%BF%D0%BE%D0%BB%D0%BD%D0%B8%D0%BC%D0%BE%D1%81%D1%82%D0%B8_%D1%84%D0%BE%D1%80%D0%BC%D1%83%D0%BB_%D0%B2_%D1%82%D0%B5%D0%BE%D1%80%D0%B8%D1%8F%D1%85

Поддерживаемые и активно развивающиеся решатели: Alt-Ergo, Barcelogic, Beaver, Boolector, CVC3, DPT, MathSAT, OpenSMT, SatEEn, Spear, STP, UCLID, veriT, Yices, Z3.

Какая из них попроще?
Аноним 31/03/23 Птн 00:15:51 102073 203
>>102052
>Анонче, помнишь, как мелким пиздюком залипал на виндовс проигрыватель, слушая тупую музыку?
я в 2007 универ закончил
иди нахуй поридж
фракталы он увидел
я фракталы в первый раз увидел в 12 лет в какой-то книжке в школьной библиотеке

>>102071
давай про представления алгебр ли что-нибудь, или, я не знаю, пучки, схемы
а не вот это всё
Аноним 31/03/23 Птн 13:14:54 102076 204
Почему про аналогии между теорией Галуа и фундаментальными группами не рассказывают в стандартных курсах? Да и вообще много про чего ещё, честно говоря, про аналогию между векторными расслоениями и проективными модулями кто-то помню упомянул на лекции, но там можно читать-учитаться, одним упоминанием не отделаешься.
Иногда такое ощущение, что про математику даже 50х годов не знаю нихуя, не смотря на профессиональную работу с чистой математикой второй десяток лет. Или я должен сам до этих аналогий дойти? Я вообще многое узнал из кулуарных обсуждений на кафедре. Сейчас с существованием всяких стэкиксченджей получше стало, но вообще большинство математических книг написано прехуёвейше.
Аноним 31/03/23 Птн 15:55:30 102077 205
>>102076
> Почему про аналогии между теорией Галуа и фундаментальными группами не рассказывают в стандартных курсах?
Какие там, собственно, аналогии?

>про аналогию между векторными расслоениями и проективными модулями кто-то помню упомянул на лекции
Это не аналогия, а теорема, причём довольно специфическая. Зачем вообще нужны вообще проективные модули? Расслоение куда более фундаментальное понятие

> Я вообще многое узнал из кулуарных обсуждений на кафедре
Это нормально. В математике очень много делается через личное общение. Конференции специально для этого существуют, например, не только твоя узкая кафедра

> но вообще большинство математических книг написано прехуёвейше.
множество существует прекраснейших книг по математике
Аноним 31/03/23 Птн 18:50:09 102079 206
>>102073
Не понимат. Есть формула вроде ((t > n AND n < c) OR (c > n)). Важен только факт, что при всех мыслимых и немыслимых значениях t, n, c она всегда выдаст тру или иногда не выдаст. Есть программа которая может проверить такой предикат на тавтологичность?
Аноним 31/03/23 Птн 20:09:15 102080 207
>>102079
такую программу можно на питоне сделать
обозначим $t > n$, $n < c$, $c > n$ через $A$, $B$, $C$ соответственно, составляем для твоей функции таблицу истинности, выводим значения $t,n,c$ при которых она даёт true или false

твоя функция не постоянная, очевидно
Аноним 31/03/23 Птн 20:49:45 102085 208
>>102080
Формула может быть любой, допустим из 200 вложенных предикатов, в свою очередь вычисляемых из 300 переменных, известно только то что она всегда даст true или false при разных значениях. SMT-решатели позволяют все посчитать автоматически?
Аноним 31/03/23 Птн 20:51:23 102086 209
>>102085
Вольфрамальфа пробовал, но она чет тупит
Аноним 31/03/23 Птн 20:55:39 102088 210
>>102085
если у неё все эти предикаты вида $t>n$, то мой алгоритм такой же
в конце концев, к сднф можно любую формулу привести
вольфрам должен это уметь
ни про какие решатели я ничего не знаю
Аноним 01/04/23 Суб 08:19:06 102090 211
>>102077
>Какие там, собственно, аналогии?
Ну вот видишь, и ты не знал. Теория этальных гомотопий. Я начинал с книжки Szamuely "Galois Groups and Fundamental Groups", после этого стали более понятны идеи в SGA1.
>Это не аналогия, а теорема, причём довольно специфическая. Зачем вообще нужны вообще проективные модули? Расслоение куда более фундаментальное понятие
"Зачем вообще нужна алгебра? Геометрия куда более фундаментальна." (inb4 "так оно и есть").
>множество существует прекраснейших книг по математике
Ты математик или погромист? Должен понимать, что утверждение "большинство математических книг написано прехуёвейше" не противоречит утверждению "множество существует прекраснейших книг по математике".

Вобщем, по твоему ответу сразу ясно, что ты тот мамкин контрариан, которому лишь бы что-то спиздануть. Напомнил мне, почему я всё меньше и меньше посщу на матхе.
Аноним 01/04/23 Суб 14:20:36 102099 212
Насколько сложная математика используется в создании современных нейросетей? Хочу конкретики, с перечислением разделов от тех, кто шарит.

Есть мнение, что нейросети это просто алгоритм, и для их создания не нужны глубокие познания в математике, а достаточно лишь прикладной программы для программистов, когда как физики и фундаментальные математики должны знать математику гораздо глубже. Это правда?
Аноним 01/04/23 Суб 15:56:20 102102 213
>>102090
>Теория этальных гомотопий. Я начинал с книжки Szamuely "Galois Groups and Fundamental Groups", после этого стали более понятны идеи в SGA1.
да, я ничего не знаю про этальные гомотопии и я не читал SGA1
оно мне прям очень надо? тогда жаль, что в университете не изучают
и до сих пор не написали нормальных книг на понятном языке, насколько я знаю

>"Зачем вообще нужна алгебра?"
по сути не ответил. проективные модули вещь достаточно специфическая, и я нигде не видел теорему свана, кроме к-теории (в которой она полезна, но вне её?)

>не противоречит утверждению
ради бога, множество прекрасных книг, хорошо написанных.
какое педанство

>Вобщем, по твоему ответу сразу ясно, что ты
сразу на личности и в оскорбления, ну что ты будешь делать

>Напомнил мне, почему я всё меньше и меньше посщу на матхе.
так и начинать очередной срач на тему "почему в вузах не учат" не особенная заслуга. ещё про детерминант вспомнил. короче, кому какое дело где ты сидишь

>>102099
>Насколько сложная математика используется в создании современных нейросетей?
умножение матриц и метод градиентного спуска
это то, что на поверхности. но думаю вряд ли что-то ещё
Аноним 01/04/23 Суб 15:57:09 102103 214
>>102102
*ещё БЫ про детерминант
Аноним 01/04/23 Суб 18:12:18 102106 215
>>102076
У Вербита в его книге по топологии есть.
Аноним 01/04/23 Суб 18:59:25 102107 216
>>102106
>У Вербита
Фу бля поход в Хибины.
Аноним 01/04/23 Суб 19:07:56 102108 217
Screenshot-675.png 40Кб, 873x177
873x177
учебник по топологии
определение топологии - на 78 блядь странице
в определении обсер
Аноним 01/04/23 Суб 19:32:04 102109 218
>>102108
почему, нормальное определение
Аноним 01/04/23 Суб 20:07:31 102110 219
>>102109
Хуита же написана, внимательнее присмотрись.
Поэтому и нужно еще десяток задачек и неделю с ними ебаться чтоб хоть что то понять. А может быть просто нормальное определение написать и несколько примеров не?
Аноним 01/04/23 Суб 20:23:19 102111 220
>>102110
напиши свое нормальное определение, мы посмотрим
Аноним 01/04/23 Суб 20:30:31 102112 221
Screenshot-675.png 45Кб, 873x177
873x177
>>102111
Хотя бы так. Потому что S не является подмножеством M, оно является подмножеством 2^M.
Аноним 01/04/23 Суб 21:11:03 102113 222
>>102112
ну да, согласен
думаю, имелось в виду, что $S$ это подмножества, но тогда пару следовало записать $(M, \{S\})$. тащемта не слишком серьёзная неточность, я считаю
Аноним 01/04/23 Суб 22:16:00 102114 223
>>102113
>не слишком серьёзная неточность
Угу, особенно когда пытаешься разобраться с чем то в первый раз по такому вот "учебнику" и у тебя уже жопа в огне.
Аноним 01/04/23 Суб 22:24:39 102115 224
>>102114
в любых учебниках есть неточности и опечатки, особенно, когда они в первой редакции. с чего начинать и по чему заниматься, это вопрос всегда откртый и не совсем однозначный

подозреваю (я его не читал), учебник вербицкого таки нацелен на тех, у кого какая-то культура математическая уже есть. ну, а упражнения всегда необходимы, и да, они помогают прояснить непонятные места тоже
Аноним 01/04/23 Суб 22:40:55 102116 225
>>102115
Только эта самая "неточность" в САМОМ БЛЯДЬ ГЛАВНОМ ОПРЕДЕЛЕНИИ.
>нацелен на тех
Да всю эту макулатуру читать можно только если ты уже знаешь минимум половину из написанного, иначе - тушите свет
>а упражнения
пустая трата времени, особенно если к ним не прилагается полноценного решения.
Аноним 01/04/23 Суб 23:26:23 102117 226
>>102116
просто начни с книжки и попроще и не страдай
"элементарная топология" виро и компании, например

>пустая трата времени, особенно если к ним не прилагается полноценного решения.
насколько я понимаю позицию миши, он считает, что упражнения, наоборот, суть самое главное. я придерживаюсь того же мнения
Аноним 01/04/23 Суб 23:35:53 102118 227
>>102117
Я придерживаюсь мнения что вы с Мишей долбоебушки. И нахуя ты лезешь со своими охуительно нужными советами, тебя о них никто не просил, так что можешь себе их сральню засунуть, ебобо.
Аноним 02/04/23 Вск 00:39:42 102119 228
>>102118
это тебя топология так травмировала?
надо запретить топологию
Аноним 02/04/23 Вск 01:49:25 102121 229
Аноним 02/04/23 Вск 16:37:32 102129 230
Какие темы из математики требуется знать в качестве базы для изучения таких вещей как:
- распознавание печатных и рукописных текстов
- распознавание объектов на видео, а также определение перемещения объектов - например что объект пересёк какую-то область и затем покинул её
- анализ текстов, исправление ошибок

Какие темы учить надо по математике для этого? За пол года реально будет изучить? Или надо 10-20 лет дрочиться?
Аноним 02/04/23 Вск 17:17:20 102130 231
Объясните, пожалуйста, гою, в чём цимес пучков?
Аноним 02/04/23 Вск 18:26:14 102134 232
>>102129
Теорию автоматов/алгоритмов/вычислимости/дискретку. Начать с мат.логики какого нибудь учебника Игошина "Математическая логика и теория алгоритмов" + там задачник. Ещё почитай несколько томов "Искусство программирования" Д. Кнута.
Аноним 02/04/23 Вск 22:13:12 102139 233
>>102130
они красивые
очень нужны на комплексных и алгебраических многообразиях, где нет разбиения единицы
Аноним 03/04/23 Пнд 00:58:55 102143 234
>>102134
За пол года получится прочитать? Или это надо десятками лет задрачивать?
Аноним 03/04/23 Пнд 07:31:57 102146 235
>>102143
Если ты будешь каждый день по 2-3 часа читать и ещё прорабатывать/нарешивать/доказывать, на протяжении 6 месяцев - то в принципе, для прикладника будет очень хорошо.
Аноним 04/04/23 Втр 15:13:52 102163 236
>>102146
Огромное спасибо
Аноним 05/04/23 Срд 16:53:52 102170 237
Кто круче матан решает chatGPT или wolfram alpha?
Аноним 07/04/23 Птн 11:11:15 102195 238
Мне кажется, что из школьной программы надо нахуй выкинуть 80% геометрии.
Аноним 07/04/23 Птн 17:48:51 102200 239
>>102195
Надо просто начать учить математике, а не той программе к которой ещё Киселёв в 19 веке учебники писал (а сейчас просто элементы "вышмата" то добавляют... То убирают...). А то школьники охуевают, когда узнают что есть множество алгебр и есть ещё какая-то супер алгебра алгебр. Под логикой они подразумевают какие-то странные афоризмы житейской мудрости и вообще не знают зачем они в геометрии что-то доказывают когда И ТАК ВСЁ ПОНЯТНО. Теория множеств для них это какая-то бесполезная херня о которой они никогда не слышали. Зато дискриминант и логарифм знают, и то ответить что он такое не смогут, как и что такое функция... Про арксинус лучше вообще не спрашивать... Сейчас попросту уже нельзя преподавать математику по этой недо программе. Она мало того что неэффективна, так ещё и вредна блять.
Аноним 08/04/23 Суб 12:25:07 102204 240
>>102200
С 1 стороны да, база хромает, мало кто понимает почему сложение столбиком работает и почему при сложении дробей нужно их приводить к общему знаменателю. Но все худо бедно могут это сделать и квадратное уравнение решить.
Но Вербит писал интересное предположение, что если выкинуть тригонометрию, алгебру и пр. и сосредоточиться на простых вещах, то уже сложение дробей для школьников будет так же тяжело, как тригонометрические уравнения сегодня. Планка упадёт. Программа для массовой школы сложный вопрос.
А вот в спец. классах определенно занимаются хуетой. И это навряд ли выйдет исправить из-за гос дроча на олимпиадки.
Аноним 08/04/23 Суб 21:51:42 102206 241
>>102170
Это ложная дихотомия, чат жпт может генерировать тексты программ в том числе и для вольфрама.
Аноним 08/04/23 Суб 21:59:03 102207 242
>>102204
Проблема с базой в математике в том, что она может основываться как на абсолютно антинаучном, идеалистическом мировоззрении, так и на научном, материалистическом и номиналистическом мировоззрении. Но чисто исторически как раз всякое мракобесие по типу Пифагора и Платона - это как раз в духе математики и математиков.
Аноним 09/04/23 Вск 07:28:38 102209 243
>>102207
>так и на научном, материалистическом и номиналистическом мировоззрении
"Однако человек нелегко оставляет предположение, основанное на чувственном представлении, что аггрегат конечных вещей, который называется миром, обладает действительной реальностью; что не существует мира, - это признается совершенно неприемлемым или, по крайней мере, гораздо менее приемлемым, чем мысль о том, что не существует бога. Полагаю - и это не служит к чести тех, которые так полагают, - что гораздо легче представить себе, что какая-нибудь философская система отрицает бога, чем представить себе, что она отрицает мир; находят гораздо более понятным отрицание бога, чем отрицание мира."
Аноним 09/04/23 Вск 12:54:01 102213 244
>>102209
>Полагаю - и это не служит к чести тех, которые так полагают
Так ты же полагаешь, еблан. Какого хуя? Быстрый гуглеж выдает
>Полагают — и это не служит к чести тех, которые так полагают
Забавно. Сначала хуйлософы понасрут кучу поноса, а потом горе переводчики еще ее отполируют как следует.
Может конечно анон писал по памяти, ну тогда мое почтение.
Аноним 11/04/23 Втр 15:04:20 102242 245
7xau1tm.png 85Кб, 751x829
751x829
В чём он не прав inb4 питау-срач? Почему тригонометрия такая убогая, но это почти никого не парит? Почему сообщество математиков - типа самых гибкоумных людей на планете - в данном вопросе ведёт себя так же инертно, как любое дефолтное стадо баранов-бюрократов? Где ещё человечеству дозволится бороться с принципом "работает - не трогай", с проклятьем обратной совместимости, тащащей за собой многолетнее нагромождение костылей, - как не в области мысленных экспериментов, где почти нет материальных рисков?
Аноним 11/04/23 Втр 16:20:50 102243 246
>>102242
Тригонометрия это мертвая наука, к современной математике не имеющая отношения. Непонятно, что с твоей точки зрения должно парить сообщество математиков, и что ты от них хочешь, чтобы они не выглядели, как стадо баранов
Аноним 11/04/23 Втр 18:46:30 102244 247
>>102243
> Тригонометрия это мертвая наука
это не наука, это устаревший инструмент типа римских цифр, который именно что мёртвый, и вместо разработки чего нового начали городить костыли на нём. Если она давно уже не про углы и треугольники, так зачем мы все формулы сводим к v3/2 и v2/2 (выведи через них синус 20градусов). Если про табличные углы, то почему только про пи/4 и пи/3, чем пи/5 и пи/7 хуже? А так в ней неуклюже сходятся измерение углов, периодические функции и аппроксимация трансцендентных чисел (полученных из целых и радикалов как правило).
> Непонятно, что с твоей точки зрения должно парить сообщество математиков, и что ты от них хочешь, чтобы они не выглядели, как стадо баранов
переписать учебники, чтоб матан и физика не были засраны ими, чтоб каждый третий интеграл не был про них и не выводился в них; а тем более всякие секансы, гиперболические и тд. Перенормировать измерение углов/тригонометрии, чтоб по наклону касательной=тангенсу можно было сразу прикинуть значение угла и наоборот и быстро вычислить. Чтоб многочлены из тригонометрических фукнций решались быстро и интуитивно. Для этого всего, конечно, есть комплексные числа через е, но они - сюрприз - сами через тригонометрию выводятся.
Аноним 12/04/23 Срд 08:15:10 102248 248
>>102244
У тебя какая-то травма. Никого синус в интеграле не задевает. Просто функция

>сами через тригонометрию выводятся.
Смотря как ввести. Можно через экспоненту по определению.
Аноним 12/04/23 Срд 09:37:27 102249 249
Вот давно уже заметил, что неосиляторы ну просто таки обожают со своей инженерной первокурсной колокольни предлагать "улучшения" для математики, отверженно борясь с заговором математического истеблишмента. То давайте заменим пи на тау, то давайте заменим эпсилон-дельта на нестандартный анализ, то запись производных\логарифов плохая, то десятичная система плохая, то синусы плохие. Сюда же добавить погромистов, требующих перейти к пруверам как к стандарту доказательств.
И это стабильно, из года в год, на протяжении как минимум 10-15 лет.
Аноним 12/04/23 Срд 14:03:57 102253 250
>>102249
>>102248
ладщно, ладно, оставим переход на личности и конспирологию, просто ответьте по пунктам на >>102242
Аноним 12/04/23 Срд 15:59:45 102262 251
>>102248
> Никого синус в интеграле не задевает. Просто функция
Если б он один был - но их дохуя, и у каждого своя запутанная алгебра взаимодействия с остальными и со своими аргументами. Представь, чтоб вместе с е ещё 5 констант комплектом шло, и они вылезали друг из друга самым непредсказуемым образом (если ты конечно не любишь зубрить формулы или каждый раз их выводить).
> Смотря как ввести. Можно через экспоненту по определению.
1.ну наверно ещё 20 способами можно. Только от геометрической наглядности (ради которой тригонометрия и затевалась) там следа не останется.
2.Если комплексное выражение удобнее, почему к нему не прибегают при первой же возможности. Я же говорю, всё начало анализа засрано тригонометрией, поэтому приходится зубрить все их 9000 тождеств. Потому что невозможно представить человека, которому бы нравилось их выводить, в отличие от тех же пределов-интегралов. Это ведро холодной кислой каши, которое тебя вынуждают проглотить перед дальнейшим
Аноним 12/04/23 Срд 16:13:22 102264 252
>>102262
ты хрень какую-то

константы $\pi$ и $e$ совершенно фундаментальные, их две, а не пять

никто 9000 тождеств не запоминает, все пользуются экспонентой, когда надо
Аноним 12/04/23 Срд 16:41:34 102267 253
RgMd37m.png 26Кб, 556x550
556x550
5Q4YoEw.png 36Кб, 638x636
638x636
>>102264
> их две, а не пять
На твоё счастье. Более того, они редко между собой пересекаются в сложных соотношениях. А тригонометрических функций далеко не две: (арк)(ко)синус, (арк)(ко)тангенс. Потом ещё появляются секасы-защеканцы и гепербролические.
> все пользуются экспонентой, когда надо
Тогда почему вот это не в экспонентах написано. Просто потому что по курсу положено сначала по-школьному делать? К чему потом приткнуть эти навыки, если ты сам сказал, что на них забивают?
Аноним 12/04/23 Срд 17:44:37 102268 254
>>102267
>Тогда почему вот это не в экспонентах написано
Написаны тождества. Для интегралов различных функций. Никто эти тождества не запоминает, всегда можно посмотреть, если надо
Аноним 13/04/23 Чтв 17:30:52 102277 255
>>10487 (OP)
Анон, верно ли что любое непрерывное отображение f из X в Y порождает отображение F между топологиями O(Y) O(X) (в обратном порядке), таким образом что F(U) принадлежит O(X) для любого U из O(Y) тогда и только тогда когда прообраз U открыт в Х? Если да, то является ли такое соответствие контравариантным функтором и что является морфизмом в категории топологий?
Аноним 13/04/23 Чтв 19:32:44 102278 256
>>102277
прообраз любого открытого множества открыт под действием непр. отобр., так что твоё F как отражение множеств O(Y) \to O(X) корректно определено. Что такое "морфизм топологий", непонятно уже на уровне объектов
Аноним 14/04/23 Птн 00:10:47 102281 257
>>102278
Объекты - топологии на множествах, морфизмы - отображения между такими топологиями. Отображения между топологиями определены тогда, когда определены непрерывные отображения между подлежащими множествами, но обратное не совсем очевидно: верно ли что каждому отображению между топологиями можно сопоставить непрерывное отображение на множествах, для которых построены эти топологии?
Если да, то свойства непрерывных отображений переносятся на отображения между топологиями. Я почему-то не встречал нигде такого упоминания, потому пытаюсь найти ошибку.
Аноним 14/04/23 Птн 10:53:28 102283 258
>>102281
>Объекты - топологии на множествах
Придётся в качестве объектов быть что-то вида "пара (X, O(X))". Тогда и морфизмы должны быть отражениями между парами

>верно ли что каждому отображению между топологиями можно сопоставить непрерывное отображение на множествах, для которых построены эти топологии?
Неверно, как легко заметить, если посмотреть на простые примеры. Скажем, пусть F:O(Y)\to O(X) переводит все элементы O(Y) в пустое множество. Такое F не отвечает никакому f:X\to Y
Аноним 16/04/23 Вск 05:25:38 102308 259
>>102268
1.Зачем смотреть, если ты сам говоришь, что экспоненциальная форма удобнее? Зачем они, а также задачи на них даны в этом и любом другом задачнике? Почему там не дадут заодно таблицу римских цифр (и лютые хитрости арифметики с ними) - ну а что, вдруг понадобятся?
2.При решении геометрических задач с многоугольниками кругами к тригонометрии всегда прибегают лишь в крайнем случае - потому что с этого момента задача необратимо уходит в алгебраическую возню с приведением (арк)углов к стандартным, чтоб решение удалось записать в радикалах и пи.
Аноним 16/04/23 Вск 16:30:20 102316 260
Почему у додиков такой бомбаж при упоминании Савватеева? Он по факту лучший математик в стране.
Аноним 16/04/23 Вск 16:46:03 102317 261
>>102283
>Неверно, как легко заметить, если посмотреть на простые примеры. Скажем, пусть F:O(Y)\to O(X) переводит все элементы O(Y) в пустое множество. Такое F не отвечает никакому f:X\to Y
Какое условие нужно добавить чтобы каждой такой F можно было сопоставить непрерывную f? Инъективность как пример, но это довольно сильное ограничение.
Аноним 16/04/23 Вск 17:15:33 102318 262
Можете попробовать по хардкору объяснить мне в чем разница между математиками и программистами? В чем у них отличается принцип мышления, подхода к решению задач, и тд?
Аноним 16/04/23 Вск 19:49:12 102319 263
>>102318
Программисты мыслят алгоритмически и пишут программы(алгоритмы). Математика же не алгоритмична в общем случае.
Аноним 16/04/23 Вск 20:06:04 102320 264
Меня интересует та задачу которую Перельман решил - она одна из 7 задач тысячелетия. Задачи тысячелетия — семь математических проблем, определённых Математическим институтом Клэя в 2000 году как «важные классические задачи, решение которых не найдено вот уже в течение многих лет», за решение каждой из которых обещано вознаграждение в 1 млн долларов США.

А почему остальные 6 не решат? Я правильно понимаю. что эти задачи не шибко то и нужны капиталистам поэтому никто их не решает ибо всем похуй?
Аноним 17/04/23 Пнд 21:03:03 102327 265
>>102320
Да, капиталисты до сих пор не захватили всю галактику и не создали автономные корабли добывающие ресурсы с планет, только потому что это не выгодно.
Аноним 18/04/23 Втр 12:40:29 102334 266
>>102327
А разве нет? Например, можно было бы с Луны тащить ресурсы, но это экономически невыгодно, проще на Земле по старинке добывать как диды.
Аноним 18/04/23 Втр 16:31:49 102337 267
>>102334
Зачем тратить земные ресурсы чтобы добыть ресурсы на Луне в минус? Ради чего?
Аноним 18/04/23 Втр 17:28:57 102339 268
>>102337
Значит я прав что те оставшиеся 6 из 7 задач тысячелетия не решают просто потому что это не выгодно корячиться решать эти задачи ради всего лишь миллионной награды?
Аноним 20/04/23 Чтв 09:52:29 102369 269
Аноним 23/04/23 Вск 17:55:45 102402 270
Сап двач надо написать реферат по теме "Вероятностно-статистический подход к обоснованию некоторых теорем геометрии." Препод всех игнорит. Можете пожалуйста дать совету, оттослать к каким-нибудь книгам и статьям, сверх буду благодарен если напишете примерный план реферата.
Аноним 23/04/23 Вск 18:09:07 102403 271
>>102402
Пока думаю начать с определения геометрической вероятности, вставить задачу из эгэ в качестве примера разобрать подводные применения теории вероятности к геометрии. Потом разобрать задачу с применением иглы бюффона для нахождения числа пи. И как я понимаю пиксельный метод это мой случай?
Аноним 23/04/23 Вск 18:15:48 102404 272
>>102249
ну записи тригонометрических функций реально хрень. я это выписывание sin и cos по 50 раз за строчку решения еще в школе возненавидел. Поэтому на черновиках всегда синус обозначал за s косинус за с.
Аноним 23/04/23 Вск 23:12:16 102408 273
Какие математические продвижения последних лет (начиная с 2019 мб) вам кажутся наиболее интересными и важными?
Аноним 23/04/23 Вск 23:12:39 102409 274
Какие математические продвижения последних лет (начиная с 2019 мб) вам кажутся наиболее интересными и важными?
Аноним 23/04/23 Вск 23:13:29 102410 275
Какие математические продвижения последних лет (начиная с 2019 мб) вам кажутся наиболее интересными и важными?
Аноним 24/04/23 Пнд 13:59:15 102412 276
>>102408
А я даже математические продвижения 2019 года до нашей эры не знаю
мимо хочу начать учить математику с самых школьных азов
Аноним 28/04/23 Птн 20:57:08 102485 277
Сап двач когда-то давно видел статью с решением Иглы Бюффона без взятия интеграла, но теперь потерял. Может кто знает или хотя бы подскажет как оно могло решатся
Аноним 07/06/23 Срд 10:15:54 102992 278
image.png 61Кб, 1668x1094
1668x1094
Есть задача.
На плоскости находится точка. Где-то ещё - прямая (точка не лежит на прямой). Нужно найти такую траекторию движения для точки, которая позволит определить, где находится прямая и при этом будет самой короткой.
Интуитивно понятно, что, вроде бы, это какая-то спираль типа изображённой на пике. Но как это доказать?
Аноним 08/06/23 Чтв 07:37:48 103010 279
>>102992
>Нужно найти такую траекторию движения для точки, которая позволит определить, где находится прямая и при этом будет самой короткой.
Перпендикуляр из точки к прямой.
Аноним 08/06/23 Чтв 07:52:26 103011 280
>>103010
Мы не знаем, где прямая. Её надо найти.
Так-то понятно, что перпендикуляр.
Аноним 08/06/23 Чтв 08:03:54 103012 281
>>103011
Прямая перпендикулярна отрезку перпендикуляра, проведённого из точки. Находится на расстоянии длины отрезка.
Аноним 08/06/23 Чтв 10:59:57 103014 282
>>103010
>>103012
Анон, ты не вкурил условие задачи.

>>102992
Что-то на стыке стохастического анализа и вариационного исчисления. Может быть релевантна статья "A probabilistic deformation of calculus of variations with constraints", а также https://en.wikipedia.org/wiki/Onsager–Machlup_function.
Аноним 08/06/23 Чтв 15:35:02 103026 283
>>102992
есть мнение, что это возможно только когда радиус монотонно и неограниченно возрастает при возрастании угла
Аноним 12/06/23 Пнд 16:54:47 103117 284
Стоит ли бросать linear algebra done right на 3 конце главы? Очень медленно идёт, почти каждое упражнение как пытка. Ни малейшего представления, где я буду это применять, насколько универсальны используемые автором приёмы рассуждения. Линал это же база баз для других разделов, а этот излагается будто ты уже на 3 курсе, и множество-топологическое мышление в тебя накрепко вбито. В общем, если учебник идёт с большим трудом, применимость не определена, лучше сразу дропать? Порванный анус гордыни как-нибудь зашью.
Аноним 12/06/23 Пнд 20:03:28 103118 285
>>103117
если тяжело, конечно, не надо
работа через силу едва ли имеет смысл
особенно учитывая, что учебников по линалу несчетное множество

но книга хорошая
Аноним 12/06/23 Пнд 23:33:43 103119 286
>>103118
>работа через силу едва ли имеет смысл
Не согласен. Лично мне только это и помогло однажды достигнуть просветления.
Аноним 13/06/23 Втр 00:44:31 103120 287
Задачи пустая трата времени

>>103118
>книга хорошая
>нет решений задач
/0
Аноним 13/06/23 Втр 08:50:03 103121 288
>>103117
Я тут с анонами не раз спорил о том, что LADR как вводная книга для рандомного самообразователя - хуита. Её идёальная аудитория - это прикладные математики (таки да, в том числе андерграды), которые будут заниматься чем-то, связанным с функциональным анализом. В крайнем случае её могут полистать особо любознательные после того, как уже пройдут курс линала по другой книжке.
Все кричат о том, что "по гауссу" и матричный дроч - это плохо. Какой-нибудь Булдырев-Павлов - это ещё одна крайность. Акслер избегает прикладных рассуждений и в то же время пропускает многие важные теоретические конструкции.
Важен баланс. Нужно дать понять, почему матрицы полезны, что за ними кроется. Дроч систем линейных уравнений или пивотов а-ля странг этого не даст, но сухое абстрактное повествование этого тоже не добьётся (тут исключаю чистых математиков).

Я бы советовал линал (и вообще любую область) читать по крайней мере по двум, разнонаправденным, книжкам.
Аноним 13/06/23 Втр 15:23:41 103126 289
>>103117
Вы, мне кажется, идиот. Весь интернет завален решениями LADR, для второго издания и вовсе есть официальный мануал от Акслера. Он также сделал accompanying видеолекции для книги. Можно также отыскать lecture notes курсов линейки из универов, вплотную идущих по LADR и не решать все подряд, а только то, что указано в problem set'ах этих самых готовых курсов.
Аноним 13/06/23 Втр 16:18:41 103127 290
>>103121>>103126
Получается из говна и палок облазив пол-интернета с горящей жопой можно собрать что-то условно удобоваримое. Почему нельзя сразу нормально сделать блядь?
Аноним 13/06/23 Втр 17:39:11 103129 291
>>103127

Потому что условно удобоваримое для всех разное.
Аноним 13/06/23 Втр 17:49:15 103130 292
Аноним 14/06/23 Срд 10:55:41 103134 293
>>103014
>>103026
Спасибо, посмотрю, что из этого можно извлечь.
Аноним 14/06/23 Срд 15:37:48 103137 294
Мне студент задал вопрос на который я, даже учитывая мои познания в комплексном анализе/теории Галуа/расширения полей/алгебрах Клиффорда, не смог дать удобоваримого для себя ответа.

Вопрос такой. Основываясь на теории алгебр Клиффорда, про мнимую единицу можно очень просто думать как об операторе поворота на $\frac{\pi}{2}$ в какой-то плоскости. К нам ещё давно на кафедру приходили ребята с теорфизики и показывали, как вообще весь электромагнетизм (и много чего ещё) можно строить без упоминания комплексных чисел - там естественным образом возникают псевдоскаляры, квадрат которых равен -1. То есть часто там, где используются комплексные числа, более фундаментальной является какая-то связь с поворотами и $SO(3)$, а точнее даже с $so(3)$. Ну ясное дело это всё изоморфно $\mathbb{C}$ или там $\mathbb{H}$, но суть в интерпретации. Это всё замечательно обобщается на н-мерное пространство, в частности становятся более очевидными всякие факты вроде того, что $SU(4)$ есть двойное накрытие $SO(6)$, если думать в терминах алгебр Клиффорда.

С другой стороны, комплексные числа - это алгебраическое замыкание $\mathbb{R}$.

Вопрос - как эти описания связаны? Есть ли какая-то фундаментальная причина связи поворотов с алгебраической замкнутостью? Что интересно, мне пару лет назад уже задавали вопрос про кажущуюся магической связь между $\Lambda^2(\mathbb{R^3})$ (грубо говоря, "площадями") и поворотами. Но там я, вроде как, сам понимаю всё хорошо.

Здесь же я студенту как-то помахал руками про фундаментальную теорему алгебры, но интуитивного объяснения я не дал. Есть ли оно?
Аноним 14/06/23 Срд 16:45:22 103138 295
>>103137
>Вопрос - как эти описания связаны
В должны быть связаны? Кватернионы же тоже про повороты, но даже поле не образуют.
Аноним 14/06/23 Срд 17:19:58 103139 296
>>103137
я, конечно, простой человек и не облдаю такими глубочайшими познаниями, как ты, но кажется очевидным, что если из прямой удалить точку, она развалится, а если из плоскости - у нас образуется гомотопический хаос,
в котором и обретает черты основная теорема алгебры. это топологический факт, а не алгебраический
Аноним 14/06/23 Срд 17:29:53 103140 297
>>103138
>Кватернионы же тоже про повороты
Так кватернионы замечательно интерпретируются в этом же самом фреймвёрке - действительные спиноры в двух измерениях это $\mathbb{C}$, а в трёх - $\mathbb{H}$. Но поскольку повороты происходят в (двумерной) плоскости даже в $\mathbb{R^n}$, то комплексные числа сами собой естественным образом появляются, даже если работать чисто над $\mathbb{R}$.
Вопрос-то мой в другом.
Пока моё объяснение - это комбинация доказательства фундаментальной теоремы алгебры через индекс точки, плюс факт того, что замкнутость релевантна только для полиномов чётной степени. Но всё равно как-то грубо выходит.

>должны быть связаны?
Годы и годы занятия математикой меня научили одному: если я не вижу связи, это скорее всего значит, что я недостаточно глубоко копаю и чего-то не знаю. Ну кроме очевидных случаев, когда объекты вообще из разных, никак не связанных областей.
Аноним 20/06/23 Втр 12:44:56 103203 298
image.png 845Кб, 1080x1304
1080x1304
Поможет ли свежая кровь вернуть мехмату былую славу?
Аноним 20/06/23 Втр 23:16:50 103209 299
>>103203
я считаю, что пусть учится каждый, кто хочет учиться,
но спиздануть про участников сво, из которых кто-то случайно убил бабушку или жену, это совсем в край ёбнуться надо
Аноним 20/06/23 Втр 23:31:47 103210 300
>>103120
тупое говно, откуда вы повылазили

>>103121
>Её идёальная аудитория - это прикладные математики
довольно странное утверждение про эту книгу, учитывая, что основное вычислительное средство в ней даётся в самой последней голове, а всё изложение построено из категорического принципа, что это средство должно быть отложено как можно дальше

>Акслер избегает прикладных рассуждений и в то же время пропускает многие важные теоретические конструкции.
одно из хороших свойств этой книги - она короткая
в ней материала не очень много, а объём (небольшой) получается, более-менее исходя из того, что всё разжёвывается крайне подробно. кому важно поместить в один том "все важные теоретические конструкции", может потискать какого-нибудь алуффи, если заняться нечем
Аноним 21/06/23 Срд 00:06:24 103211 301
>>103210
Типичный любитель порешать задачки и высерающий
>топология на множестве есть множество его открытых множеств
чтд мозгов нет нихуя
Аноним 21/06/23 Срд 08:49:20 103216 302
>>103210
>довольно странное утверждение про эту книгу
Утверждение перестанет быть странным, если его дочитать до конца, а не цитировать, оборвав на половине.
Аноним 21/06/23 Срд 10:02:37 103217 303
>>103216
я читал её всю, правда, давно и не помню уже деталей, кроме собственных моих впечатлений от неё (в частности, мне лично очень помогла)
мне непонятно, в каком месте можно посчитать, что "Её идёальная аудитория - это прикладные математики", по-моему, это чепуха
Аноним 21/06/23 Срд 11:31:43 103218 304
>>103217
Анон, в оригинальном посте сказано - книга для прикладных математиков, связанных с функаном. Ты вырвал из контекста и прицепился к части утверждения, и это полностью исказило смысл. И даже во второй раз ты не понял, о чём речь.
Ну так-то конечно всё будет чепуха, если читать посты жопой. Как у тебя с русским вообще? "Если его дочитать до конца" очевидно относится к утверждению, а не к книге.
Аноним 21/06/23 Срд 13:13:06 103219 305
>>103218
ну ок, я не особо представлаю себе, что такое прикладные математики, связанные с функаном, чем они разительно отличаются от остальных математиков, которых ты от них отделяешь, и чем книга якобы для одних идеальна, а для других типа нет. с учётом того ещё, что книга вообще для начинающих и её задача - не рассказать досконально материал, а произвести только введение в него, такое разделени выглядит ещё более странно. но ради бога
Аноним 21/06/23 Срд 13:44:04 103224 306
>>103219
>я не особо представлаю себе
Раз не представляешь, то зачем так оживлённо спорить? Очевидно, что у тебя какое-то однобокое представление.
Функан и его приложения вроде матфизики - это самая естественная среда для понимания определителя в терминах спектра, как делает Акслер.
Собственно сам Акслер, да и другие математики, писавшие книги в том же ключе (Булдырев/Павлов, скажем) - все писали статьи про функан. Вот так совпадение! (нет)

Если чего-то себе не представляешь, то может лучше посидеть-подумать, почитать, поискать. Если ты правда не видишь разницы между подачей линала через матрицы и по гауссу, через функан-ориентированную линзу вроде Акслера и Булдырева-Павлова, и через общую алгебру и теорию модулей.
Аноним 21/06/23 Срд 14:24:09 103227 307
>>103224
>Очевидно, что
если тебя кто-то не понимает или понимает как-то не так, это не значит априори, что он идиот

>все писали статьи про функан.
давать материал можно по-разному, однако я не могу солгаситься с тем, что подача одним образом более важна для какой-то специализации, как бы ты ее не называл, чем другая подача, коль скоро речь идет о вещах совершенно базовых и нужных для всех. из перечисленных тобой подходов они нужны все и всем (хотя модули я бы отложил), но это не значит, что они должны содержаться в одной книге, а если не содержатся, то книга не годится

>Если чего-то себе не представляешь, то может лучше
что за любовь все время поучать
Аноним 22/06/23 Чтв 13:59:19 103232 308
>>103227
>что за любовь все время поучать
Что за любовь нести ересь вроде
>коль скоро речь идет о вещах совершенно базовых и нужных для всех. из перечисленных тобой подходов они нужны все и всем

Линал используется инженерами, экономистами, социологами. Им совершенно не нужно знать, что такое кольцо эндомофризмов, для того, чтобы пользоваться матрицами. Им совершенно не нужно знать, что такое спектр оператора. Им просто нужно знать, как эти вещи используются для решения конкретных задач. Например, экономист может просто кликнуть в интерфейсе статпакета и получить результат критерия коинтеграции или стационарности в виде собственных чисел. Знать, что там за ними "реально" кроется - ненужная для них информация. Также, как для пользования смартфоном не нужно знать квантовой механики, которая используется под капотом.

А уж если мы сузим круг читателей до математиков, то тогда книга Акслера вызывает ещё больше вопросов, потому что внешней алгебры нет, а определитель не упоминается на протяжении почти всей книги.
Аноним 23/06/23 Птн 14:36:40 103246 309
>>103232
>Что за любовь нести ересь
ересь говорит твой собеседник или нет - это твоё личное мнение о том, что ты от него слышишь. в то время как манера поучать - это явное свойство личности (твоей собственной). странно сравнивать одно с другим

>то тогда книга Акслера вызывает ещё больше вопросов, потому что внешней алгебры нет, а определитель не упоминается на протяжении почти всей книги.
это вводная книга для начинающих. про внешнюю алгебру (а заодно и более основательно про определитель) начинающий прочтёт в следующей книге
Аноним 24/06/23 Суб 08:41:50 103255 310
>>103246
>это вводная книга для начинающих
>про внешнюю алгебру (а заодно и более основательно про определитель) начинающий прочтёт в следующей книге
А если и в той книге не будет, то начинающий прочтёт об этом в третьей книге...
Аноним 24/06/23 Суб 19:56:51 103260 311
>>103255
может быть, и в третьей, а почему нет? вообще я нахожу это не особо умным - критиковать книгу за то, чего в ней нет, когда следовало бы обсудить, что в ней есть. у книги акслера не очень аккуратное название (понравилось оно ему), но в целом понятно (должно быть), что рассказать весь линал - это явно не ее цель. на самом деле она дает альтернативный подход к самым на базовым понятиям, стараясь раскрыть их геометрический смысл, и надо обладать весьма узким вглядом на вещи, чтобы считать, будто такой подход полезен только будущим специалистам по функану. ну, когда у человека в смартфоне под капотом используется квантовая механика (что правда лишь частично), такой его взгляд наверно можно понять
Аноним 26/06/23 Пнд 10:32:40 103275 312
>>103260
твой пост ушёл в сторону от изначальной критики анона
ну с подменой тезиса конечно легче спорить, что тут скажешь, это ж двощ
Аноним 26/06/23 Пнд 22:03:16 103292 313
>>103275
если кратко, его критика сводилась к тому, что эта книга полезна только будущих специалистов по функану, а для других не полезна, поскольку в ней нет внешнего произведения (чего ещё в ней нет, пояснено не было). По моему мнению (подробности см. выше) первое есть ерунда, а второе бессмыслица. Если я что-то важное упустил, можешь указать
Аноним 27/06/23 Втр 11:58:05 103303 314
>>103292
ну то есть читаешь ты жопой, ясно понятно
в оригинальном посте ни слова про внешнее произведение не было, кстати
и то, что она ни для кого больше не полезна, тоже сказано не было
>ерунда
>бессмыслица
ну если самому придумывать себе утверждения, с которыми спорить, то таки да, всё будет ерундой и бессмыслицей
к нам ccс/сай/ что ли утекает
Аноним 27/06/23 Втр 13:05:21 103307 315
>>103303
а ты настырный.
хорошо, я пройдусь по основному посту, хотя в ясности его не упрекнёшь (у автора свои собственные определения "прикладных математиков" и т.д.)

>LADR как вводная книга для рандомного самообразователя - хуита.
абсолютно прекрасна как вводная вводная книга для рандомного самообразователя: в ней ясно и подробно разжёвывается геометрическая природа линала, что в вводных книгах нечасто встретишь (здесь приходит на помощь преподаватель, которого у самозанятого обучающегося нет), это чудесно и очень помогает пониманию.

>Её идёальная аудитория - это прикладные математики
глупости, книга даёт альтернативный взгляд на вещи с акцентом на их геометрическую природу; это полезно всем, кто изучает предмет. кроме того, альтернативный взгляд на вещи особенно полезен математикам, которые доказывают теоремы (не будем вдаваться в детали терминологии "прикладные математики"); всегда здорово указать, как теоремы доказываются другими способами

>в то же время пропускает многие важные теоретические конструкции
(такие как внешнее произведение, было пояснено ниже)
это нестрашно для вводной книги

> Нужно дать понять, почему матрицы полезны, что за ними кроется
матрицы - это операторы, насколько я помню, в этой книге было сказано об этом. если нет (я уже не помню), то это недостаток, тут я соглашусь
Аноним 27/06/23 Втр 13:11:48 103308 316
>>103307
и ещё

>Акслер избегает прикладных рассуждений
я не знаю, что такое "прикладные рассуждения"
Акслер даёт альтернативные рассуждения - тем, что более традиционны. это прекрасно; про традиционные рассуждения читатель прочтёт в другой книги. вообще, нет ничего плохого в том, чтобы читать разные книги (наоборот - хорошо)
Аноним 27/06/23 Втр 21:38:23 103328 317
Почему закрепилось точнее, не открепилось говорить "по модулю", а не по остатку? Типа чтоб заумней звучало и гнобить тех, кто перепутал с модулем, который абсолютное значение?
Аноним 27/06/23 Втр 22:08:49 103330 318
>>103328
в англ говорят “modulo”
модуль называют “absolute value”
Аноним 28/06/23 Срд 22:16:24 103348 319
>>103330
Культ Карго какой-то?
Аноним 28/06/23 Срд 22:21:31 103349 320
>>103328
>Типа чтоб заумней звучало
Как и половина научных словечек. Себя не похвалишь - никто не похвалит. Не выставишься умным - посчитают дураком.
Аноним 01/07/23 Суб 15:36:29 103418 321
Решаю листочки с задачами (Пойа, Правдоподобные рассуждения в математике), до этого ходил в НМУ, но ничего не чувствую.. Как будто просто решаю задача за задачей, нет повышения мудрости. Что делать? Или так и задумано?
Аноним 02/07/23 Вск 00:20:12 103435 322
>>103418
>нет повышения мудрости
Ну это естественно же.
Вот ты собираешься решить задачу. Возможны два варианта:
1) ты либо знаешь как ее решить - и только зря тратишь время
2) ты не знаешь как ее решить - и ничего не решишь
Это на столько блядь очевидно... Как только удается задачаблядям засирать нубасам мозги.
Аноним 02/07/23 Вск 17:45:58 103449 323
16654323443.jpg 204Кб, 785x1101
785x1101
>>103418
А для меня они чет ЖОССССКИЕ (не все, но многие). Сам пока сижу вспоминаю чего учил в вузе по mathprofi
Аноним 02/07/23 Вск 22:34:50 103452 324
Хочу обмазаться математической эзотерикой. Что курить?
Аноним 02/07/23 Вск 23:27:08 103454 325
>>103435
скажи честно, тебя с первого курса отчислили? или ты недобрая баллов на егэ, и тебя не взяли на матфак?

И теперь ты мстишь аноним за свою неудачную жизнь, как настоящий двачер. Теперь ты будешь неосилятор-петух

Прошу всех нормальных анонов запомнить; когда этот шиз снова влезет, можно будет применять. Предлагайте также свои варианты, если этот не очень
Аноним 03/07/23 Пнд 00:41:09 103457 326
>>103454
У безмозглой задачебляди подгорает от очевидных фактов.
ЕГЭ я не сдавал кстати потому что его еще тогда не придумали, малолетний отброс.
Аноним 03/07/23 Пнд 00:57:52 103458 327
>>103457
А, т.е. ты в том возрасте, когда мозги уже в принципе не соображают, особенно в том, что хоть немного отличается от того, к чему они привыкли, и потому не можешь решать задачи. Тебе больно даже читать их условия, не говоря уже о том, чтобы остановиться на какой-нибудь и подумать

Ты понимаешь, что уже никогда не восполнишь те знания, которые вовремя не осилил, не получишь уже образование и ничего уже не достигнешь, из-за этого страдаешь и мстишь анонам, которые только начинают. Из зависти и от бессилия
Аноним 03/07/23 Пнд 01:11:47 103459 328
>>103458
Нет, просто делюсь по доброте душевной своей житейской мудростью приобретенной и отшлифованной с годами, чтобы аноны хуйней не страдали.
Ты же, долбоебушка малолетний, можешь хоть за яйца себя подвешивать, мне не жалко.
Аноним 03/07/23 Пнд 06:42:02 103463 329
>>103459
ой, какие мы снисходительно щедрые

И как же анон определит, где «мудрость», а где больной шизобред, больше похожий на троллинг? Что ж, иногда полезно делать выбор самому. К тому же в случае неосилятора-петуха ответ, по-моему, очевиден
Аноним 03/07/23 Пнд 14:00:56 103467 330
>>103463
Ну да, те у кого есть мозги разберутся где "троллинг" а где очевидные вещи. Малолетние безмозглые ебанько вроде тебя пусть дальше головой об стену бьются, мне не жалко.
Аноним 04/07/23 Втр 20:02:09 103517 331
Хуя вы тыт все снобы. Тройной интеграл хоть без листочка с ручкой возьмёте, перельманы?
Аноним 04/07/23 Втр 21:58:14 103520 332
>>103517
если интеграл берется, с ним с большой вероятностью справится компьютер
Аноним 04/07/23 Втр 22:48:39 103521 333
image.png 25Кб, 328x359
328x359
>>103520
>с ним с большой вероятностью справится компьютер
Здесь же доска для ментатов, какие такие разумные машины?
Аноним 06/07/23 Чтв 11:47:14 103544 334
>>103521
несешь какую-то дичь
Аноним 06/07/23 Чтв 20:44:35 103567 335
>>103520
Какпонять что интеграл берется?
Аноним 06/07/23 Чтв 22:01:57 103569 336
>>103567
если компьютер не взял, то и ты не сможешь

Ещё, если интеграл расходится, то это обычно не очень трудно доказать, но это другая задача
Аноним 06/07/23 Чтв 23:15:28 103573 337
>>103521
Че с матмеха выперли?
Аноним 06/07/23 Чтв 23:17:31 103574 338
>>103452
Начни с трилогии Ромы Михайлова
Аноним 07/07/23 Птн 11:53:18 103583 339
>>103569
В теорфизике интегралы компьютерами вообще не берутся, но физики их как-то берут
Аноним 07/07/23 Птн 12:30:52 103584 340
>>103583
Физики крутые вообще, но они же физики, а не математики. Так что за ними не сюда
Аноним 07/07/23 Птн 12:57:47 103587 341
>>103583
>В теорфизике интегралы компьютерами вообще не берутся
Монте-карло в помощь. Руками никто ничего не берёт. Если аналитически не берётся машиной, то он машиной берётся численно.
Аноним 07/07/23 Птн 13:06:19 103588 342
>>103587
Умеют ли машины хорошо работать с обобщёнными функциями в интегралах?
Аноним 07/07/23 Птн 18:07:58 103602 343
>>103587
Ключевое слово - теорфизика.
Аноним 07/07/23 Птн 20:17:45 103611 344
Screenshot-672.png 432Кб, 1920x1080
1920x1080
Хотел бы я посмотреть как компудахтеры справляются вот с такими интегралами. Правда.

Даже с банальным преобразованием Фурье там надо что то подкрутить чтобы вольфрамальфа не срался под себя.
Аноним 07/07/23 Птн 20:40:56 103614 345
а ты как с ними справляешься? см. внимательнее>>103569
Аноним 07/07/23 Птн 21:31:10 103623 346
>>103614
Тупорылый школотун тупорыл во всем. Народная мудрость.
Аноним 10/08/23 Чтв 09:48:11 105378 347
Репост вопроса.
Ищу книжки про разложение модулей через идемпотенты/инволюции в контексте алгебр Клиффорда и спиноров. Удивительно мало литературы об этом, в большинстве учебников про это вообще не говорится, или уходят не в ту степь (Book of Involutions). Пока самое релевантное, что я нашёл, это "Clifford Algebras and Spinors".
10/08/23 Чтв 11:29:41 105382 348
2+2*2?
Аноним 10/08/23 Чтв 11:44:51 105384 349
>>105382
질문은 낱말안에 물어야 한다
Аноним 16/08/23 Срд 06:45:39 105686 350
image.png 824Кб, 1060x1280
1060x1280
Аноним 16/08/23 Срд 08:41:52 105693 351
Напишите мне список что мне нужно изучить чтобы я смог хотя бы немного начать понимать уравнения Максцелла
Аноним 16/08/23 Срд 11:55:38 105713 352
>>105693
Если на стандартном инженерном уровне, то обычный матанализ.
Если в контексте теорфизики, то нужно почитать про дифформы, а ещё лучше про алгебры Клиффорда.
Аноним 22/08/23 Втр 01:44:12 106269 353
Аноним 27/08/23 Вск 14:17:44 106614 354
Анон-антисоветчик-педагог-психолог, который здесь обитает, кинь ссылок на резерч по IQ у разных профессий, от меня в благодарность как обычно.
Аноним 27/08/23 Вск 15:46:20 106617 355
изображение2023[...].png 5Кб, 285x28
285x28
>>10487 (OP)
Анон, что это вообще за конструкция, если R это не кольцо, а поле, а R(fin) это не идеал, а кольцо? В этом контексте R это поле гипервещественных чисел, R(fin)- конечные гипервещественные числа.
Аноним 27/08/23 Вск 15:47:43 106618 356
>>106617
Разметка подвела, R* поле, R(inf) кольцо.
Аноним 27/08/23 Вск 16:01:12 106619 357
>>106617
Вероятно тебя просят указать размерность векторного пространства, так что $\mathbb{R}_{\ast}\mathbb{R}_{\text{fin}}$ нужно рассматривать как факторпространство по подпространству.
Аноним 27/08/23 Вск 16:02:53 106620 358
>>106619
*$\mathbb{R}^{\ast}/\mathbb{R}_{\text{fin}$
Аноним 27/08/23 Вск 16:07:19 106621 359
>>106619
Спасибо, в предыдущих примерах все объекты были кольцами, так что я и не подумал про факторпространства. Чем в таком случае будет R*/R(fin)? Очень похоже на R.
Аноним 17/09/23 Вск 22:15:33 108406 360
Нужна помощь зала. Что я упускаю, что вы могли бы мне посоветовать?

Читаю определения по матеше и нихуя не понимаю.
Подозреваю у себя СДВГ, по линии бабки была биполярка.

Даже если у меня СДВГ и внимание блуждает и я улетаю в фантазирование/ как мне это компенсировать то блядь??

Стараюсь высыпаться и прямо ощущаю огромный буст до уровня 50% от нормы (обычно 10-20).
Как не пытайся мне требуется МНОГО времени чтобы вникать в определения, что хотел сказать автор учебника.
Не имеет значение смотрю я ролик популярный на ютубе или это учебник. Все равно буду тупить.

Проходил тест на дискалькулию и выдало что у меня ее нет (7/50). СДВГ много баллов набираю в любом тесте.

У меня нет проблем с визуализацией объектов, но есть проблема с визуализацией концепций типа даже ебаных дробей чтобы интуитивно вырабатывать логику решения если я уже знаю как их вычитать.

Это какая-то трабла с рабочей памятью. Я не могу ее хакнуть. Даже если записывать что я недопонимаю то забуду что я делал в предыдущих шагах пока записывал. Даже сейчас проебал мысль пока писал предыдущее предложение. Вспоминаю... Вспоминаю... 3 minutes later... а вот мысль... что я даже эти вопросы не могу вовремя нормально сформулировать пока усваиваю материал. Оперативка мозга попросту перегружена.

Или читаю определение и вспоминаю каждое ебаное слово - что оно значит. Мозг еле подгружает. Могу даже на ебучем "знаменателе" затупить, вспоминать пару секунд что это и картина начинает трещать так как я забываю остальное где нахожусь пока усваиваю.

Я ощущаю математические концепции как "прозрачные", как какой-то хаос. Потому не могу их визуализировать. Проходил визуальные тесты и мой результат был до 10 объектов (восстанавливаешь порядок), то есть я вписывался в норму в 7 объектов. Спокойно могу визуализировать физические явления. С физикой проблем не было, кроме этих ебаных определений снова.

Таких проблем нет когда я уже знаю предмет. То есть ресурс мозга не уходит на бесполезные домыслы что подразумевалось.

Хули делать без фармы? Закладки и индусов не предлагать.
Аноним 18/09/23 Пнд 09:56:55 108422 361
>>108406
"Не шарить в математике" - это стильно, модно, молодёжно. Если сказать, что ты безграмотный, то на тебя косо посмотрят. Если сказать, что ты не знаешь, где Антарктида, то тоже в общем-то рукоплесканий не последует. А вот если сказать, что ты никогда не мог в математику, у тебя родители вообще художники, ну нет таланта, генетически хуёмоё, adhd, дискалькулия, то тебя соучуствующе похлопают по плечу, скажут, мол, у нас у всех также бро, кому она вообще нужна эта маняматика, мы творческие люди да ещё биполярОЧКА и нужна таблетОЧКА для адхд.
Десятилетиями непонимание математики обществом нормализовывалось. Всё скидывается на таинственную генетическую предрасположенность, "другой склад ума", ADHD, и прочие деструктивные мемы вроде "таланта". ADHD это вообще пик современной моды. "Я что-то читал на тилибоне а потом посмотрел в окно и отвлёкся, эх вот ADHD не даёт нам покоя ребзя правда?". "Я делал что-то долгое нудное джва часа подряд и устал, ну у меня точно ADHD же верно??".

Из моей личной практики преподавания, ровно 100% учеников и студентов, жалующихся, что они "никогда математики не понимали", просто-напросто проебали основы ещё в школе. Скажем, идёт тема производных, там примеры основаны на понятиях их тем предыдущих лет вроде функций, графиков, тригонометрии, алгебраических выражений, и т.д.
Школьная математика - крайне иерархичная вещь. В какой-нибудь истории или литературе можно проебать, Скажем, 7-8ой класс, и нормально закончить 9ый. В математике это не проходит. Во всяком случае в школе - например, ты можешь начать читать линейную алгебру и общую алгебру (кольца\модули, поля и теория Галуа), полностью пропустив весь курс матанализа.

Поэтому "помощь зала" следующая:
1) Хватит искать генетического козла отпущения. Если только у тебя буквально нет медицинской справки о серьёзных нарушениях деятельности головного мозга, то ты можешь понять когомологии и спектралки, можешь выучить китайский язык, можешь научиться играть на пианино, можешь научиться рисовать и программировать, коль скоро ты вложишь время и усилия.
2) Если тебе не понятны, скажем, дроби, то сформулируй точно - что именно тебе не понятно. Задавай вопросы как себе, так и кому можешь вокруг (преподаватели, сверстники). Качай рефлексию.
Аноним 20/09/23 Срд 02:53:54 108533 362
>>108406
>Читаю определения по матеше и нихуя не понимаю.
Люди разные.

У некоторых людей есть эйдетическое воображение - очень детализированное и богатое. Например, воображая ёлку, такие люди могут пересчитать на ней все иголки. А у некоторых людей ситуация полностью противоположная - визуального воображения нет вообще. Такие люди в принципе не понимают, как это - вообразить визуально что-нибудь. Математические определения часто рассчитаны как раз на таких людей. На людей, у которых визуальное мышление либо в принципе отсутствует, либо может быть выключено произвольно, по желанию.

Это ярко проявляется при изучении общей топологии. Некоторые люди при слове "шар" не могут не воображать себе нечто вроде трёхмерного шарика для пинг-понга. И если таким людям предложить рассмотреть вложенную последовательность пятимерных шаров в пространстве с какой-нибудь хитрой метрикой - эти люди перегреются и поломаются. Они, читая текст со словом "шар", не могут не воображать шарик. У них это не отключается.

А людям, которые умеют при слове "шар" не воображать геометрическую картинку, но при этом вызывать в памяти все интуиции, связанные с понятием шара, изучать общую топологию будет очень легко. Ровно до тех пор, пока не понадобится построить какой-нибудь контрпример, хе-хе.

Возможно, чтобы тебе было проще работать с определениями, тебе стоит развить навык: при чтении текста отключать автогенерацию в воображении всяких ненужных ассоциаций. Визуальных и не только.
Аноним 20/09/23 Срд 07:37:00 108538 363
>>108533
>У некоторых людей есть эйдетическое воображение
>А у некоторых людей ситуация полностью противоположная - визуального воображения нет вообще.
Эта "теория" была опровергнута ещё несколько десятилетий назад.
>На людей, у которых визуальное мышление либо в принципе отсутствует
Такого не существует, если только у тебя нет крупных отклонений, скажем дополнительной хромосомы, или опухоли мозга.
Аноним 20/09/23 Срд 14:47:48 108588 364
>>108538
Это просто факты.
Аноним 20/09/23 Срд 16:54:17 108599 365
>>108588
Тебе в обос/сай/ с такими "фактами".
Аноним 20/09/23 Срд 19:58:30 108606 366
Аноним 20/09/23 Срд 21:40:44 108613 367
>>108606
>They found that 0.8% of the population was unable to form visual mental images, and 3.9% of the population was either unable to form mental images or had dim or vague mental imagery
Аноним 20/09/23 Срд 22:03:48 108615 368
>>108613
Подозрительно похоже на процент математиков, нет?
Аноним 21/09/23 Чтв 00:36:05 108624 369
>>108613
Чего сказать то хотел?

Мне реально не понятно как можно что то "видеть в голове". Если все так умеют, то почему если попросить схематически нарисовать велосипед большинство людей рисуют невнятную поебень, ведь им достаточно было бы вызвать его образ в своем мысленном взоре и перерисовать.
Аноним 21/09/23 Чтв 07:38:42 108628 370
>>108606
>>108613
С математическими способностями это не связано никак.
Аноним 21/09/23 Чтв 10:34:16 108634 371
>>108628
Судя по биографиям математиков и личным беседам, связь есть.
Аноним 21/09/23 Чтв 11:45:07 108640 372
>>108634
присоединяюсь к анону выше, тебе с такой "наукой" в /sci/, там твои братья по разуму
Аноним 21/09/23 Чтв 12:10:45 108642 373
>>108615
Да, а ещё на число детей с дцп или синдромом дауна
Аноним 21/09/23 Чтв 14:23:17 108652 374
>>108642
А это не одни и те же люди?
Аноним 05/10/23 Чтв 17:13:44 109342 375
Здравствуйте, уважаемые форумчане. Кто-нибудь может, пожалуйста, подсказать решение, или раздел главу параграф чего-нибудь, задача следующая:

Есть набор чисел (показания датчиков, которые должны быть равны между собой) (12,84; 12,99; 13,07; 13,1; 12,49; 12,56)
Каждое из этих значений является Х(допустим, X - среднее из набора)/Y(число в районе 10000 плюсминус пару десятков).
Цель минимум: определить Y для каждого из чисел набора
Цель максимум: определить Y для каждого из чисел набора на основании нескольких таких наборов чисел (срезов показаний датчиков).
Аноним 13/10/23 Птн 09:05:15 109734 376
IMG3619.JPG 110Кб, 1280x933
1280x933
Привет, посоветуйте, что читать по теории чисел.
Аноним 31/10/23 Втр 16:38:04 110334 377
image.png 19Кб, 1082x190
1082x190
image.png 324Кб, 769x265
769x265
image.png 89Кб, 819x887
819x887
>>10487 (OP)
Здравствуйте, перекачу свой пост из sci:
Читаю Челпанова логику, почему из 64 комбинаций силлогизмов он называет верными только 11, игнорируя IEO? Это глава XV "Силлогизм. Фигуры и модусы силлогизмов".
Пик1 - в экселе накидал таблицу, где сначала я закрасил темно-серым все комбинации которые противоречат восьми правилам силлогизмов. Потом зеленым закрасил те комбинации которые автор называет верными. Как результат IEO - остался не закрашенным, то есть он и ни одному правилу не противоречит, но и не приводится автором как верный.
Пик2 - фрагмент из учебника где перечисляется список верных силлогизмов.
Пик3 - доказательство того, что я не шиз и некий "Ratigan" на древнем форуме уже задавался этим вопросом, но ему так и не ответили
Аноним 31/10/23 Втр 19:42:33 110339 378
image.png 299Кб, 757x587
757x587
>>110334
Отмена, он в той же главе объясняет почему не берется IEO хотя он говорит, что оно противоречит четверотому правилу, но я еще не понял почему
Просто почему-то уже после того как объяснил фигуры
Аноним 31/10/23 Втр 22:14:46 110343 379
>>110339
Хорошо, что ты нашёл ответ сам, потому что никто бы тут не стал самостоятельно разбираться в твоих обозначениях и определениях, которые ты не предоставил (и даже если бы предоставил, то это всё равно это к тематике доски имело бы не большее отношение, чем шахматы, например, или лингвистика).
Аноним 01/11/23 Срд 09:52:11 110346 380
>>110334
>логику
Не математика.
Аноним 01/11/23 Срд 10:14:34 110348 381
>>99999
>>100000
Аноним 01/11/23 Срд 12:35:03 110349 382
>>110339
> Burbara, Celarent, ...
Учите блядь нормальную математическую логику по современным учебникам, нахуй вы в этом древнем говне ковыряетесь.
Аноним 02/11/23 Чтв 15:34:13 110377 383
MHv1RRqiJkA.jpg 74Кб, 403x592
403x592
>>110349
>Учите блядь нормальную математическую логику
Мир не ограничивается логикой предикатов, даже мир математики.

>нахуй вы в этом древнем говне ковыряетесь.
Я лично сейчас читаю свежее, прикл, пока ещё только в начале. Очень интересно как автор вводит различие на сущностные и не-сущностные термы, для того чтобы обосновать модальную силлогистику. Всё выглядит вполне понятно и ясно. Берётся знаки из aeio, берётся знак из XNQM, и вот у нас к примеру NXN силлогизм Barbara:
A aN B - Всё А необходимо принадлежат B
B aXC - Всё B принадлежат C
A aN C - Всё А необходимо принадлежат C
Значки Q и М обозначают это двухсторонняя возможность или односторонняя возможность. Берёшь два терма, формируешь копулу (связку) и ставишь два терма субъект и предикат. Тут самый сок в семантике которая обосновывает почему такая-то фигура работает, мне нравится.
Аноним 02/11/23 Чтв 15:36:12 110378 384
>>110377
Ты доской ошибся.
Аноним 02/11/23 Чтв 15:49:13 110379 385
16945398599150.jpg 273Кб, 493x739
493x739
>>110378
>Ты доской ошибся.
Аноним 02/11/23 Чтв 19:35:59 110383 386
>>110377
Вроде бы для таких любителей щекотать очко придумал модальную логику.
Аноним 03/11/23 Птн 03:48:29 110385 387
фрг1.png 92Кб, 1125x686
1125x686
фрег.png 71Кб, 1137x560
1137x560
ШтрихФРг.png 24Кб, 1124x599
1124x599
>>110383
Неее. Ну так модальность была у Аристотеля, но в новое время из-за падения схоластики уже закрепилась традиционная логика (которая суть урезанная силлогистика Аристотеля). Потом был Фрёге который срал Буля за отсутстиве кванторов и что его "запись в понятиях"(которые предикаты) через штрих (прикл3) гораздо удобней чем тупой закос под арифметику (прикл1), паралелльно разбирая насколько Пеано база (прикл 2). А уже потом после, сложилась традиция оформившая классическую математическую логику как пропозициональную и предикатную логику. И как-раз где-то в этот периуд началось создание "не-классических логик", хотя "Классическая логика" - на тот момент существовала совсем ничего. То-есть, S1-S3 модальные логики появились в 1930 годы, хотя ещё лет 30 назад Фрёге не получил никакого признания как и его исчисление (На лекции Фрёге было буквально 0 человек, то-есть, исчисление с предикатами нахуй никому не было нужно вплоть до начала 20 века когда Рассел подметил парадокс и началась движуха с аксиоматизацией). Крч, странно тыкать пальцем во всё что не является матлогикой и говорить что это что-то странное, когда сама матлогика взлетела из-за того что звезды так сложились.
Аноним 03/11/23 Птн 06:07:57 110387 388
>>110379
Обпучкался с пикчи. Охуенно просто.
Аноним 06/12/23 Срд 22:28:06 111318 389
photo2023-12-06[...].jpg 245Кб, 959x1280
959x1280
photo2023-12-06[...].jpg 192Кб, 960x1280
960x1280
photo2023-12-06[...].jpg 208Кб, 960x1280
960x1280
photo2023-12-06[...].jpg 188Кб, 960x1280
960x1280
Кому-нибудь было бы интересно забрать что-то из этого?
Аноним 06/12/23 Срд 22:29:05 111319 390
photo2023-12-06[...].jpg 170Кб, 960x1280
960x1280
photo2023-12-06[...].jpg 205Кб, 1280x959
1280x959
Аноним 07/12/23 Чтв 01:11:03 111324 391
>>111318
прекрасные книги, куда же ты их так

я бы взял половину, если бы был в дс
Аноним 07/12/23 Чтв 13:52:44 111331 392
0e60fda511bb294[...].jpg 22Кб, 429x600
429x600
>>111324
Мне не хотелось отдавать. Но вот случилось одно небезызвестное решение верховного суда, к сожалению.
Аноним 07/12/23 Чтв 19:44:31 111337 393
28/12/23 Чтв 14:01:18 111666 394
image.png 24Кб, 528x474
528x474
вот такое траливарирование всех математиков с моей стороны

@FORMAL_V
Аноним 28/12/23 Чтв 18:15:56 111670 395
а
а
Аноним 23/01/24 Втр 22:24:16 112126 396
image.png 249Кб, 547x362
547x362
>>10487 (OP)
Матаны, а есть ли не вещественные математические константы? Если точно нет, то чем комплексные, кватернионы и прочие покемоны хуже?
Аноним 24/01/24 Срд 07:04:08 112127 397
Аноним 24/01/24 Срд 07:23:17 112128 398
>>112127
Можно вспомнить и другие базисы, но почему так скудно то? Неужели на прямой больше интересных чисел с интересными свойствами чем на плоскости, например.
Аноним 25/01/24 Чтв 20:54:41 112150 399
> Престиж математики в Китае был высок. Каждый чиновник, чтобы получить назначение на пост, сдавал, помимо прочих, и экзамен по математике, где обязан был показать умение решать задачи из классических сборников.

В I—V вв. н. э. китайцы уточняют число
�\pi — сначала как
10
{\sqrt {10}}, потом как 142/45 = 3,155…, а позже (V век) как 3,1415926, причём открывают для него известное рациональное приближение: 355/113.

В это время китайцам уже было известно многое, в том числе:

вся базовая арифметика (включая нахождение наибольшего общего делителя и наименьшего общего кратного);
действия с дробями и пропорции;
действия с отрицательными числами (фу), которые трактовали как долги;
решение квадратных уравнений.
Был даже разработан метод фан-чэн (方程) для решения систем произвольного числа линейных уравнений — аналог классического европейского метода Гаусса.[2] Численно решались уравнения любой степени — способом тянь-юань (天元术), напоминающим метод Руффини-Горнера для нахождения корней многочлена[3].

В области геометрии им были известны точные формулы для определения площади и объёма основных фигур и тел, теорема Пифагора и алгоритм подбора пифагоровых троек.

В III веке н. э., под давлением традиционной десятичной системы мер, появляются и десятичные дроби. Выходит «Математический трактат» Сунь-Цзы. В нём, помимо прочего, впервые появляется задача, которой позднее в Европе занимались крупнейшие математики, от Фибоначчи до Эйлера и Гаусса: найти число, которое при делении на 3, 5 и 7 даёт соответственно остатки 2, 3 и 2. Задачи такого типа нередки в теории календаря.

Другие темы исследования китайских математиков: алгоритмы интерполирования, суммирование рядов, триангуляция.


А я и не знал, что китайцы такими продвинутыми были/есть. Кто знает может и американские индейцы имели хоть какие-то представления о математике дальше арифметики.
Аноним 25/01/24 Чтв 21:08:28 112151 400
>>112126
нули зета-функции Римана
Аноним 25/01/24 Чтв 23:29:34 112152 401
>>112150
>были/есть
Только были. В конечном итоге только европейцы смогли непрерывно пронести научную традицию и развить современную науку в 18-19 веках.
Аноним 29/01/24 Пнд 03:37:39 112181 402
Почему дзета функция дефолтно записывается как ряд дробей, а не натуральных, просто знак в степени поменят? Выглядит же проще, базовей наверно.
Аноним 29/01/24 Пнд 05:14:12 112182 403
>>112152
Европейцы знали Евклида и маняфантазировали по образу Начал строить и другие разделы матеши. В итоге взлетело благодаря Лагранжу. А так европейская матеша ничем особо не выделялась до Нового времени.
Аноним 29/01/24 Пнд 16:52:18 112191 404
>>112182
>Нового времени
Ну да, о нём и речь.
Аноним 29/01/24 Пнд 23:25:40 112196 405
>>112181
как тебе нравится, так и записывай
Аноним 30/01/24 Втр 11:14:50 112199 406
>>112196
я спрашиваю не как мне записывать, а почему в мире, исторически так принято
Аноним 30/01/24 Втр 22:32:06 112210 407
>>112199
нету никакого соглашения или традиции относительно того, как записывать слагаемые вида $1/x^k$, в виде дроби или в виде $x^{-k}$; если ты везде видишь дроби, это только оттого, что авторы выбирают такой способ записи
Аноним 22/02/24 Чтв 23:02:51 113106 408
Toby Hendy.png 9090Кб, 2736x2736
2736x2736
Где искать nerd тянок?
Которые знают и изучают такие науки как математика и физика?
https://www.youtube.com/watch?v=Zw5t6BTQYRU
https://www.youtube.com/watch?v=Zw5t6BTQYRU

Не серьезно? Разве такие в тиндерах ведутся?
В библиотеку идти и как додик пытаться их домогаться пока они заняты изучением науки? Так себе, по моему библиотека последнее место для этого.

Идти в университет и в столовой подкатывать? Охрана/полиция сразу прийдет, причем не обязательно потому что тянка с которой пытался знакомиться, а просто кто-то заметит что пытаешься в университете будучи не студентом знакомиться.

Так что где? Какие интересы у таких тянок? Разве что шахматный клуб? Что еще?

p.s. я sapiosexual, и были исследование которые показывали что у умных людей мозг более развит и больше wrinkles/folds in the brain, которое безусловно передастся потомкам.
Аноним 22/02/24 Чтв 23:20:29 113108 409
>>113106
логарифмы вроде в школе проходят, можешь там поискать
sage Аноним 22/02/24 Чтв 23:23:06 113109 410
>>113108
не надо троллить, к тому же ты не умный тролль, ведь смог бы разузнать что это была ссылка на одну из самых простых тем на которую она преподает
Аноним 23/02/24 Птн 00:31:14 113111 411
>>113109
>к тому же ты не умный тролль
не хочешь, получается, со мной размножаться? жаль
>ссылка на одну из самых простых тем на которую она преподает
не волнуйся, всё остальное тоже покрывается в школьной программе, ну или в пту каком-нибудь в крайнем случае
Аноним 23/02/24 Птн 09:58:21 113113 412
>>113106
>Идти в университет и в столовой подкатывать? Охрана/полиция сразу прийдет
придет дружинник-студотрядовец с фсзкультурного факультета и объяснит, что так делать низя
Аноним 23/02/24 Птн 12:55:11 113114 413
>>113113
ты математик? где водятся nerd females?
где их искать и знакомиться?
Аноним 24/02/24 Суб 01:08:05 113128 414
>>113114
я самопровозглашенный юморист, а свои амурные предметы продолжайте обсждать без меня
Аноним 24/02/24 Суб 14:16:09 113138 415
>>113106
Университетские "nerd females" - это обычно пиздец.
Во-первых, у них охуевшее количество внимания даже по женским меркам, потому что тут их совсем меньшинство => завышенное ЧСВ и армия фанатов. Готов ли ты через это пробиваться?
Другая проблема - няшнотян тут еще меньше, остальные - серые мыши и/или шизухи. Недавно читал в чате общаги, как один парень жаловался, что соседка принципиально не смывает говно за собой.
P.S. - в тиндерах водятся.
Аноним 25/02/24 Вск 08:24:02 113176 416
Привет, пожалуйста помогите. Ищу решебник по статистическим задачам всяким околоматематическим. В доступе только сборники задач для колледжей-вузов, а решебников нет. Где искать?
Аноним 25/02/24 Вск 10:31:43 113178 417
hqdefault.jpg 10Кб, 480x360
480x360
>>113176
>Ищу решебник по статистическим задачам всяким околоматематическим
Это как спросить, есть ли решебник по механике сплошных сред, задачам всяким околоматематическим. Ответом будет, собственно, сам учебник по механике сплошных сред. Необходимо, как в привиденном мной примере, так и в твоем случае, искать учебник а не решебник. Как вообще должен в твоем представлении, выглядеть решебник по разным статистическим задачам? Сам подумай, если это приложение теории (а чем иным может быть задача?), а тебе нужен некоторый решебник этих задачь (то-еесть решение приложений), то наверное тебе наверное здесь нужен прикладной учебник в интересующей тебя области приложения, где изложение статистики соседствует с решением типовых задачь из области приложения. Вот тебе и задачник и решебник одновременно. Если ты имел в виду под "околоматематическим" и "статистическим" нечто совершенно иное, то наверное стоило бы уточнить это иное, а не описывать свою проблему одним единственным предложением.
Аноним 25/02/24 Вск 10:44:11 113179 418
>>113178
По школе есть решебники, должны быть и здесь. Только как и знания никто их давать не собирается. Я аутировать и сидеть не намерен. Это нецелесообразно в моей ситуации. Это предмет статистика.
Аноним 25/02/24 Вск 11:13:25 113180 419
бгг.jpg 191Кб, 830x436
830x436
>>113179
>Только как и знания никто их давать не собирается. Я аутировать и сидеть не намерен. Это нецелесообразно в моей ситуации.
Аноним 25/02/24 Вск 11:29:10 113181 420
>>113180
Может хватить демагогии? Я был в 2 школах, там только 1 парень хорошо учился, потому что не кушал говно и жил в средних условиях в рамках города, да еще и фамилия заканчивается на штейн, еще и отец программист старой закалки. А хуету мне собачью рассказывать не надо с детьми алкашей я обязательно стану умным.
Аноним 25/02/24 Вск 12:09:34 113182 421
>>113179
Должны? Кому должны, тебе?
Решебники к университетским учебникам можно пересчитать по пальцам одной руки. Нет такого массового запроса, как в школе.
В лучшем случае в некоторых вузах студентота складирует свои решения для будущих поколений во всяких облаках, но туда левому человеку попасть почти нереально.
Аноним 25/02/24 Вск 12:21:27 113184 422
>>113182
Да мне пох уже, не хотите не надо. Мигранты красавчики постепенно все ниши займут, местная диспора будет за бесплатно обучать.
Аноним 25/02/24 Вск 12:28:40 113185 423
>>113184
Да-да, шиз, это не решебников не существует - это от тебя их ПРЯЧУТ и не хотят давать из вредности.
Аноним 25/02/24 Вск 18:36:22 113204 424
>>113180
Но справедливости ради второй вариант это куча разных широко применимых идей и легко эти три книги можно навернуть за месяц. В то время как первый - это узко-специализированный дроч, причем минимум полгода надо чтобы нормально разобраться.
Аноним 27/02/24 Втр 22:40:13 113271 425
>>113204
Нет никакой справедливости. Есть у кого бабло и репетиторы для них остальные шизы вот и все.
Аноним 28/02/24 Срд 14:04:59 113314 426
>>113271
Ты шиз не потому что у тебя нет бабла, а потому что не понимаешь различия между школьным и университетским образованием, вместо этого считая что от тебя что-то СКРЫВАЮТ.
В школе мало учебников и есть унифицированная программа, поэтому там очень легко сделать решебники.
В универе полнейшая солянка - каждый препод вправе читать практически как захочет и активно этим пользуется. Даже в рамках одного факультета и одного предмета разные потоки могут учиться на разных задачниках. Более того, очень часто в универах учатся по своим собственным методичкам, которые являются сборником задач из десятка разных задачников, а то и придуманных из головы. Кто и как должен решебники делать в такой ситуации? И главное для кого - для сотни-другой студиков? Есть буквально пара общепризнанных учебников, у которых есть массовая аудитория, сравнимая со школьной - это например Демидович, для которого есть решебник. В остальных случаях, особенно если предмет узконаправленный, ни у кого нет просто ресурса и мотивации делать решебники.
Поэтому делают не решебники, а разборы типовых задач, что тебе и советовали уже выше.
Еще ты можешь загуглить записи семинаров по статистике.
Но ты, конечно же, предпочтешь загнать правацкую телегу про мигрантов.
Мимо без бабла и репетов, всю жизнь на бюджете
Аноним 28/02/24 Срд 22:59:51 113333 427
>>113314
пидараха или блатняк вся суть
Аноним 29/02/24 Чтв 08:46:13 113345 428
Репост из параллельного треда.
Ищу книжку по алгебре, в которой будет рассказано, как в произвольной ассоциативной алгебре искать подалгебры Ли через инволютивные антиавтоморфизмы. Общая картина понятна (если α
- такой морфизм, то ${a \in A: \alpha(a)=-a}$ образуют алгебру Ли), но хочется детально.

Ну и ещё вдогонку: есть ли где-то переводы (можно на англ) французских статей Арнольда? Особенно интересует "Sur la géométrie différentielle des groupes de Lie de dimension infinie et ses applications à l'hydrodynamique des fluides parfaits".
>inb4 французский там не сложный
Очень хорошо, значит точно кто-то уже перевёл.
Аноним 29/02/24 Чтв 11:45:46 113347 429
>>113345
>Особенно интересует
Перевода, скорее всего, нет, но по идее хотя бы часть материала должна быть в его книжке по топологическим методам в гидродинамике.
Аноним 29/02/24 Чтв 12:54:54 113351 430
>>113347
Даже и забыл, что у него такая книжка есть, спасибо анон!
Но французский в статье и правда не сложный, так что наверное прочту и её. Ещё нашёл блог пост Тао об уравнении Эйлера-Арнольда, там тоже много интересного.
Аноним 29/02/24 Чтв 13:42:43 113353 431
>>113345
>но хочется детально
Посмотри литературу/разделы учебников по классификации полупростых алгебр Ли, не совсем то, что ты спрашиваешь, но может что-то полезное найдешь.
Аноним 29/02/24 Чтв 15:14:35 113367 432
>>113353
>по классификации полупростых алгебр Ли
Видимо да. Вот буквально на днях прочитал ту самую статью Дынкина, "Структура полупростых алгебр Ли". Он там с ничего (ну, с линала) всё выводит, включая корневые системы и свои диаграммы. Очень занятно, и это он в 23 года написал. Но всё равно не то, что мне нужно - ну или по крайней мере я слишком тупой, чтобы понять, как это связано. Так что читаю сейчас про разложение Картана.
Ещё нашёл The Book of Involutions, там тоже много интересного.

А вообще конечно нужно просто взять и прочитать нормальный учебник. Взял плохую привычку таскать факты из разных мест, а потом над ними сидеть думать джва года.
Аноним 01/03/24 Птн 02:22:06 113419 433
>>113345
Посмотри первую главу Xu, Representations of Lie Algebras and Partial Differential Equations.
Аноним 01/03/24 Птн 10:27:36 113428 434
834658965282.png 36Кб, 1116x198
1116x198
>>113419
Хоть что-то есть, спасибо анон.
>пик
хмммм
Аноним 02/03/24 Суб 03:03:13 113466 435
Screenshot2024-[...].jpg 133Кб, 1399x393
1399x393
Подскажите аналог формулы Бернулли, ту же хрень, но не для ровно m раз, а не для m И БОЛЕЕ раз.
Аноним 02/03/24 Суб 12:20:28 113472 436
>>113466
Блин, случайно лишнее "не" написал…
Крч, правильно так:
Подскажите аналог формулы Бернулли, ту же хрень, но не для ровно m раз, а ДЛЯ m И БОЛЕЕ раз.
Аноним 02/03/24 Суб 19:39:34 113477 437
https://www.youtube.com/watch?v=-3hGewm8qho
Какой же уебищный сука пидорас понасрал своих What is... говновидосов на все темы с абсолютно нулевым содержанием.
Аноним 04/03/24 Пнд 10:49:17 113526 438
Котаны, есть тут кто разбирается в гамильтоновой механике? В контексте классической (симплектической) механики, наверное, потому что в квантмехе по-моему это менее актуально.
Вопрос, если что, про математику. Какая фундаментальная причина того, что преобразование Лежандра вообще полезно (в контексте теорфизики)? Фактически мы переходим от N диф уравнений 2-ого порядка на многообразии к 2N диф уравнениям 1-ого порядка на кокасательном расслоении.
То есть уже сразу есть два преимущества: ур-я 1ого порядка проще, и на кокасательном расслоении "богаче" структура.

Я вот долго думол, и мой ответ такой: переход к уравнениям 1ого порядка позволяет динамику интерпретировать как потоки/локальные группы диффеоморфизмов (симплектоморфизмов). И тут сразу вся геометрия возникает, группы/алгебры Ли, симплектическая геометрия, гамильтоновы потоки, скобка Пуассона, и т.д.
Ведь с системой ур-й 2-ого порядка такого не добиться, верно? Без конвертакции её в эквивалентную систему ур-й 1ого порядка.
Аноним 04/03/24 Пнд 12:18:42 113528 439
>>113526
про преобразование Лежандра я не помню ровным счётом ничего, но там была какая-то достойная геометрия за ним

попробуй посмотреть Ж. Лере, Лагранжев анализ и квантовая механика.
я не обещаю, что там будет, но возможно что-то и найдётся
Аноним 04/03/24 Пнд 15:10:33 113529 440
>>113526
Преобразование Лежандра устанавливает двойственность между формализмами Лагранжа (вариационное исчисление) и Гамильтона (симплектическая геометрия).
Смотри: Годбийон, Дифференциальная геометрия и аналитическая механика; Tulczyjew, The Legendre transformation; da Silva, Lectures on Symplectic Geometry; https://ncatlab.org/nlab/show/prequantized+Lagrangian+correspondence#HamiltonianTrajectoriesAndPrequantizedLagrangianCorrespondences
Аноним 04/03/24 Пнд 18:37:41 113535 441
>>113529
Это чатжпт сгенерил? Ты и не сказал нихуя. Очевидно из сообщения, что он знает, что это такое.

>>113526
Найди общие черты описания механики и термодинамики. И почитай Marsden или Арнольда.
Аноним 04/03/24 Пнд 18:52:08 113536 442
>>113535
Был вопрос, почему преобразование Лежандра фундаментально полезно. Ответ - это преобразование устанавливает двойственность между двумя формализмами классической механики. Всё, что он написал, следует из этого, и подробно описано у того же Годбийона.
Аноним 06/03/24 Срд 13:43:18 113595 443
>>113536
Нет
"Устанавливает двойственность" это на уровне чатжпт, который не понимает, что за двойственность, и что вообще за задача и предметная область, но просто где-то прочитал, что там двойственность
Аноним 06/03/24 Срд 14:18:13 113597 444
1627435591717.png 107Кб, 1788x284
1788x284
>>113595
Круто, но я просто воспользовался формулировкой Годбийона.
Аноним 06/03/24 Срд 14:34:22 113598 445
>>113526
По мне так вся эта симплектическая геометрия высосана из хуя.
>мы переходим от N диф уравнений 2-ого порядка на многообразии к 2N диф уравнениям 1-ого порядка
переход основан на потрясающем факте что производная 2ого порядка это производная 1го порядка от производно 1го порядка.
>преобразование Лежандра вообще полезно
думаю что то там со свойствами производных и как они изменяются при этом преобразовании, как например преобразования Фурье сводят дифференцирование к алгебраическим манипуляциям, возможно там что то похожее есть
Аноним 06/03/24 Срд 15:50:28 113599 446
>>113598
>По мне так вся эта симплектическая геометрия высосана из хуя.
сильное утверждение. чувствуются глубокие основания под ним и знание предмета

>преобразования Фурье
совершенно другое. преобразование Фурье - чисто аналитическая вещь, существует только в $\mathbb{R}^n$ (есть попытки обобщить на римановы многообразия, но они выглядят ужасно)

преобразование Лежанра по сути своей геометрическое, оно вектора (элементы векторного расслоения) переводит в ковектора (элементы двойственного расслоения), которые действуют специальным образом
Аноним 06/03/24 Срд 17:44:15 113603 447
>>113599
Хуй соси, губехой тряси, чмондель.

Two differentiable functions ... are said to be Legendre transforms of each other, if their derivatives are inverse functions of each other
https://ncatlab.org/nlab/show/Legendre+transformation

Какой с этого профит - хз
Аноним 06/03/24 Срд 18:19:35 113604 448
>>113603
петух, опять треды перепутал
Аноним 06/03/24 Срд 18:48:36 113605 449
>>113604
Я буду радостно срать тебе в рот во всех тредах.
Аноним 06/03/24 Срд 19:29:50 113607 450
>>113605
>радостно срать
Хоть что-то у тебя получается
Аноним 06/03/24 Срд 19:36:04 113608 451
>>113607
Рад что тебе по нраву сглатывать мое говно.
Аноним 06/03/24 Срд 20:40:12 113609 452
Здарова мужики. Подскажите чему равно число i? В инете чёто не нашёл. Хочу пароли крутые сделать. Типа из математических чисел. Типа пи, число е, золотое сечение и i (мнимая единица). Пока эти нагуглил, но может ещё какие-то математические числа есть. Типо чтобы если забыл пароль можно было в калькуляторе подсмотреть. Плюс так придётся их учить на изусть. Это потом поможет в изучении математики. Все числа нашёл чему равны, а вот i не понятно. Типа пи это 3.1415 и тд. Это я уже выучил. А сколько будет i если его тоже числами написать?
Аноним 06/03/24 Срд 21:01:29 113611 453
>>113609
>А сколько будет i если его тоже числами написать?
в десятичной системе не представимо
>Хочу пароли крутые сделать. Типа из математических чисел.
Это слабые пароли.
Аноним 06/03/24 Срд 21:10:29 113612 454
>>113611
>в десятичной системе не представимо
Мне в любой пойдет. Главное чтобы можно было цифрами напечатать>>113611
>Это слабые пароли.
Знаю. Фильтры на сайтах обычно просто цифры не разрешают как пароль использовать. Мне это туда где нужно цифры вводить. Типа пароль для разблокировки телефона или пин к приложению банка
Аноним 06/03/24 Срд 22:39:03 113613 455
>>113608
ты сам наглатываешь собственное говно

>>113609
число $i$ равно числу $i$, других записей для него бывает немного. иногда его записывают $j$, иногда $\sqrt{-1}$ (подразумевается запись числа, а не вычисление функции)
Аноним 06/03/24 Срд 23:27:56 113614 456
>>113613
Поздно маневрировать, говноглот.
Аноним 07/03/24 Чтв 16:11:28 113628 457
Как избавиться от синдрома самозванца?
Я аспер в хорошем месте, выступаю на конференциях, есть несколько статей с научруком. Но мой вклад в эти статьи 10-15%, и хоть все говорят что это норма, но я все равно чувствую себя тупым. Как от этого избавить блять
Аноним 07/03/24 Чтв 17:01:19 113630 458
>>113628
был на твоём месте, но не поборол синдром самозванца
но я вообще пребываю в тяжёлых комплексах всю жизнь

>Но мой вклад в эти статьи 10-15%
достаточно, если тебе в них 100% понятно

ещё не переставай любить свою работу и не переставай никогда учиться, стремиться узнавать новое. дальше будет видно
всё, что могу посоветовать
Аноним 07/03/24 Чтв 18:21:42 113640 459
>>113630
Чмондель, так ты ж реально тупой как дрова, какой нахуй синдром.
Аноним 13/03/24 Срд 04:19:40 113685 460
Пусть у нас есть ассоциативная алгебра A над $\mathbb{R}$ с 1. Может ли быть такое, что $ab=b$ для $a, b \in A$ отличных от 1? То есть, свободно ли "действие" умножения?
Аноним 13/03/24 Срд 12:04:13 113689 461
>>113685
Возьмём в качестве A поле вещественных чисел, в качестве a -- любое ненулевое вещественное число, а в качестве b -- число 0. Тогда соотношение ab=b выполняется.
Аноним 13/03/24 Срд 12:05:28 113690 462
>>113689
>ненулевое
Неединичное, конечно.
Аноним 13/03/24 Срд 14:27:19 113694 463
>>113689
>>113690
Да, я неточно сформулировал. А если что-нибудь нетривиальное?
Аноним 13/03/24 Срд 15:36:57 113695 464
>>113694
И чтобы без делителей нуля
Аноним 13/03/24 Срд 18:36:44 113698 465
>>113695
Условие, что ab=b, эквивалентно условию (a-1)b=0. Как тут без делителей нуля?
Аноним 05/04/24 Птн 13:25:16 114151 466
А вообще вот что интересует. Кто-то в матеше пробовал вычислять(интерпретировать, трактовать) логические сложения?

К примеру яблоко и груша, чья трактовка это еда или салат. В том смысле, что вычислять абстрактное какого-то яблока или же конкретность абстрактного того же яблока, что само есть абстрактное. (Еда как абстрактное, салат как блюдо, где блюдо есть конкретное еды, но абстрактное яблока).
Аноним 05/04/24 Птн 14:24:46 114153 467
>>114151
Что такое абстрактное, что такое конкретное
Аноним 05/04/24 Птн 14:36:05 114154 468
>>114153
Хм. Ну вообще, если говорить о том, что еда есть абстрактное яблока, а яблоко есть конкретное еды, то это можно воспринимать как множества. Ну думаю тут ясно, только правда есть одно но, у множества {яблоко, груша} есть название еда, и это тут тоже важно.

Но вообще как хочешь трактовать можно. Хоть равенством. Еда это яблоко, еда это груша, еда это еда, яблоко это груша, но груша не яблоко.

Так что я бы сказал бы, наверное, что это любое бинарное отношение. Вроде порядковым называлось.
Аноним 05/04/24 Птн 14:37:06 114155 469
>>114153
Только правда есть условие, по сути, что абстрактное это логическое сложение, а конкретное это элемент логического сложения.
Аноним 06/04/24 Суб 18:00:52 114174 470
>>113698
>>113685

Значит нужны структуры без вычитания.
Предлагаю покопать в сторону тропического полукольца.
Тут как раз под рукой бумажка, где наводят линейную алгебру над полукольцом (которое уже над R), так что ,в зависимости от определений, ответ найден

https://www.imprs-mis.mpg.de/fileadmin/imprs/imprs-ringvorlesung-2018_may-22.pdf
Аноним 06/04/24 Суб 18:01:18 114175 471
>>113698
>>113685

Значит нужны структуры без вычитания.
Предлагаю покопать в сторону тропического полукольца.
Тут как раз под рукой бумажка, где наводят линейную алгебру над полукольцом (которое уже над R), так что ,в зависимости от определений, ответ найден

https://www.imprs-mis.mpg.de/fileadmin/imprs/imprs-ringvorlesung-2018_may-22.pdf
Аноним 07/04/24 Вск 12:02:41 114184 472
>>114155
Каким образом тут появляется дизъюнкция
Аноним 10/04/24 Срд 16:40:56 114236 473
Если 1+2+3...=-1/12 в чисто условном смысле, связанном с какими-то там дззета-хуета-функциями,- то зачем там поставили именно знак равенства, а не другой? У него же просто другая смысловая составляющая. Это выходит дешёвый софизм ради кликбейта, наплели пурги на ровном мести.
Аноним 10/04/24 Срд 19:49:31 114264 474
>>114236
Равенство то там настоящее. Другой смысл у знака $\sum$, который понимается НЕ как предел частичных сумм. И то смысл не другой, а обобщенный. Ведь везде, где обычная сумма даст число, эта обобщенная сумма даст такое же число. Просто там, где привычный ряд скажет "не определено" обобщенный даст число
Аноним 16/04/24 Втр 00:21:33 114363 475
>>114264
> Другой смысл у знака ∑ , который понимается НЕ как предел частичных сумм.
тогда вернее сказать, что другой смысл в принципе у всех сумм бесконечных последовательностей? Если в результате операции предельного перехода они дают конечное число - их можно привести к бытовому уровню понимания. Если не дают - то это абстрактная хрень, обращаться с которой нужно аккуратно. И числовое значение им можно присвоить не предельным переходом (который даёт в лучшем случае бесконечность), а другими операциями (которые в свою очередь могут включать в себя предельные переходы сходящихся рядов наверно).
> Просто там, где привычный ряд скажет "не определено" обобщенный даст число
так в том и >50% дела, что в этой сумме нет неопределённости, тут есть предел +бесконечность. Не определено - это +1-1+1-1..., например. А в этом выражении если впереди поставишь ещё число или поменяешь порядок конечного числа членов, оно поведёт себя совсем не как сумма, выдаст не те числа, что ожидаешь.
Аноним 16/04/24 Втр 05:38:17 114368 476
>>114363
Тут есть определенность, если жить на $\mathbb{R}\cup\{+\infinity,-\infinity\}$, что делает матан гораздо более громоздким, да и вообще это не поле. Над обычным R проще жить, и там предела нет.

Перестановка конечного кол-ва элементов не изменит значение суммы ряда, ни в обычном, ни по Чезаро. Вероятно есть на этот счет теоремы и для любого обобщенного суммирования рядов, но сходу я ручаться не стану.
Аноним 16/04/24 Втр 11:22:54 114370 477
>>114368
Да, от перестановки конечного кол-ва элементов ряд не поменяется в силу линейности взятия суммы ряда и того факта, что любой ряд с конечным колвом ненулевых элементов сходится.
Аноним 16/04/24 Втр 12:29:54 114372 478
У меня орейро. Листал Пескина-Шредера и пытался найти кое что в индексе. Короче, оказывается они слова перевели, а НОМЕРА СТРАНИЦ в нем взяли прямо из английской версии. Горжусь россиянской смекалочкой.
Аноним 18/04/24 Чтв 22:35:14 114425 479
Я только сейчас, решая легкую задачку по теории меры, понял, почему прообразы сохраняют операции на множествах, а образы - нет. Просто прообразы это отображения между булеанами, у булеанов есть естественная структура чумов, а пересечения и объединения в чумах это универсальные конструкции.
Аноним 19/04/24 Птн 02:20:11 114429 480
>>114425
Это ты так таблицу истинности => описал?
Аноним 19/04/24 Птн 02:48:55 114430 481
>>114429
Не понял, к чему ты это, таблица истинности импликации в этом контексте это про множества отображений между множествами с не более чем одним элементом.
Аноним 19/04/24 Птн 13:53:49 114446 482
>>114425
>сохраняют операции на множествах
Что значит сохраняют?
Аноним 19/04/24 Птн 15:44:27 114448 483
1571598397280.png 22Кб, 532x228
532x228
Аноним 22/04/24 Пнд 16:23:18 114529 484
>>114264
-1/12 - это значение анал продолжения дзетафункции в -1, а если другая анал-функция в данной точке будет представима этим рядом, то может она дать другое значение? Или она будет тождественно равна первой в своей области определения?
Аноним 22/04/24 Пнд 17:14:19 114532 485
>>114529

Аналитическое продолжение одной и той же функции единственно.
Аноним 22/04/24 Пнд 17:33:12 114533 486
>>114529
-1/12 можно получить и по-другому, например разложив $\sum n\exp(-\alpha n)$ в нуле в ряд Лорана, и взять оттуда константу.
Аноним 25/04/24 Чтв 03:29:11 114563 487
Появляются ли сверхбольшие числа типа грэма или трее(3) где-то помимо комбинаторики? Например, возможно ли представить, что нечетные совершенные числа, исключения из сильной гипотезы Гольббаха, нетривиальные нули начинают попадаться где-то там? Или тот факт, что такие числа далеко за пределами любой интуиции, как-то связан со сложностью задачи (некомбинаторной), т.е. если в этой задаче фигурируют такие большие числа, то она почти абсолютно непробиваема?
Аноним 25/04/24 Чтв 21:18:22 114580 488
>>114563
Что такое большие числа? Функция бобра от эквивалента одного мегабайта состояний это большое число?
Аноним 26/04/24 Птн 01:40:09 114596 489
>>114580
Ничего удачнее пока предложить не могу, чем два варианта:
-числа, размер которых невозможно оценить без применения рекурсии типа стрелочной нотации и тд. Ну т.е. максимум разрешено перемножать степенные башни типа 10^10^10... Даже если тратить все ресурсы и всё время до тепловой смерти вселенной на постройку компьютера, который их считает и держит в памяти, нифига не приблизишься.
-перемножение самых маленьких вероятностей, связанных с реальными физическими параметрами и оценками, и взять обратное от этого числа. Что типа из флуктуаций возникнет не больцмановский мозг, а суперкластер галактик или юпитер вдруг полетит прямо к солнцу, туннелирует через него, вернётся на орбиту и покажет ему фак. На это ж всё не строго нулевая вероятность, верно? Опять же построить максимально большой комп, который не сделается черной дырой и пусть он считает максимальное время, которое отводят более-менее общепринятые физические теории (большой разрыв, тепловая смерть или что там ещё).
Если всё это для тебя звучит как хуйня, то тогда прямо и чисто произвольно: числа, бОльшие, чем число Грэма. Можно взять даже поменьше: там же 64 итерации со стрелками надо провести, пускай будем считать с 30 итерации. И допустим, постоянная Эйлера-Маскерони таки рациональна со знаменателем такого размера. Означает ли это, что доказать её рациональность тогда почти невозможно?
Аноним 29/04/24 Пнд 02:04:21 114637 490
1714345460342.jpeg 455Кб, 1438x1438
1438x1438
Внимание! Ищу математика, который готов иногда отвечать на мои математические вопросы в ЛС. Часто пытаюсь что-то изобретать, на практике встречаются всякие интересные и уникальные задачки, а спросить не у кого. Взамен могу предложить то же самое по программированию.
Тг: vronu
Аноним 29/04/24 Пнд 14:20:48 114648 491
>>114637
>Взамен могу предложить то же самое по программированию.
Полезность сомнительна. Математикам помощь в программировании чаще всего не требуется. Большинство, конечно, не знает, как правильно писать "корпоративный" код по шаблонам, или какие там самые новые фреймворки, но базовые вещи уж всяко известны (по крайней мере, двощерам), скажем написать скрипт в пейтоне, или поднять сервер.

Вообще, выросши на irc и форумах, никогда не понимал этого. "Вот мой тг", "вот дискорд". Ну ты же блядь вот сейчас сидишь на доске, ну хули не спрашивается-то? Зачем приплетать ещё какую-то мокрописку, для этого не предназначенную. Тут даже латех прикрутили.
Аноним 29/04/24 Пнд 16:38:40 114661 492
>>114648
Вообще, требуется. Но программисты такую помощь называют фрилансом и просят денег. Мне бы например один алгоритм по-нормальному на суперкудахтере бы запустить, а скилов не хватает.
Аноним 29/04/24 Пнд 19:12:48 114663 493
>>114648
Потому что:
1. Я засру доску, либо наоборот, мои сообщения утонут в ней
2. Доску не проверяют так часто, как мессенджеры
3. Обращение к человеку лично работает лучше, чем обращение к массе, потому что в последнем случае легко проигнорировать вопрос
4. Иногда хочется уйти немного в оффтоп или даже во что-то личное
5. Я пробовал и нихуя мне никто не ответил
Аноним 29/04/24 Пнд 19:14:10 114664 494
>>114661
Нет, я же сказал, помощь за помощь. Примерно как у друзей.
Аноним 30/04/24 Втр 08:56:55 114681 495
>>114663
>Я засру доску, либо наоборот, мои сообщения утонут в ней
Скилл ишшью
>Доску не проверяют так часто, как мессенджеры
Если кто-то постоянно проверяет мессенджеры, то его умственные, в том числе математические, способности уже ставятся под сомнение.
>Обращение к человеку лично работает лучше, чем обращение к массе, потому что в последнем случае легко проигнорировать вопрос
Иди со своим НЛП куда пришёл.
>Иногда хочется уйти немного в оффтоп или даже во что-то личное
Ну естественно причина оказалась не в математике, а в социоблядских потребностях.
>Я пробовал и нихуя мне никто не ответил
Игнорировал правила борды, вопрос был тупой или скучный, мало ли причин.
Аноним 03/05/24 Птн 17:55:32 114720 496
Почему формула поиска простых чисел основана на степени двойки? Почему со степенью тройки не состряпают или другого простого числа? Может, поэтому совершенных нечётных не могут найти, что с двойкой хорошо поработали, а с тройкой нихуя?
Аноним 04/05/24 Суб 04:25:41 114732 497
Аноним 25/05/24 Суб 14:28:18 115170 498
Как самому(с нуля, ну может и нет) вкатиться в какое-нибудь финансовое моделирование/прогнозирование
Аноним 25/05/24 Суб 17:04:28 115176 499
>>115170
Нуль бывает разный. Финансовое моделирование тоже. В какой-нибудь Renaissance Technologies отбирают людей из PhD по чистой математике, добившихся успехов в науке. У нас в универе финансовую математику преподают тем, кто раньше собирался теоретической физикой заниматься какой-нибудь. Но есть и намного проще пути, я уверен. И это не отдельные случаи, в квантах (quantitative finance) очень любят сильны математический бэкграунд и вроде бы он даже пригождается временами.
Самое важное, что нужно знать, насколько я понимаю, это статистические методы. Для этого нужно как-то первый курс мат.анализа понимать (ну или calculus, более приложения-ориентированная дисциплина), потом теорвер, потом уже собственно статистика. Ещё дифуры, кажется, нужны. Это примерно год учебной программы.

Ну и по классике дата анализ и машинка, это сейчас основные инструменты в области. Тут линейная алгебра сильно нужна.

Насколько нужно знать экономику и как её учить — вообще без понятия.
Аноним 25/05/24 Суб 17:05:37 115177 500
>>115176
Алсо добавлю, что я описал, какие навыки нужны, слышал это от других людей, сам не занимаюсь этим. Другое дело, что для вката на рынок, возможно, нужна другая стратегия.
Аноним 02/06/24 Вск 19:43:52 115306 501
Посоветуйте литературы по кольцам Джекобсона, желательно где теория развивается без использования леммы Нетер о нормализации и где лемма Зарисского и теорема Гильберта о нулях доказываются как следствия теории.
Ответить в тред Ответить в тред

Check this out!

Настройки X
Ответить в тред X
15000
Добавить файл/ctrl-v
Стикеры X
Избранное / Топ тредов